48578900 gre physics 8677 solutions

124
DRAFT The Physics GRE Solution Guide GR8677 Test http://groups.yahoo.com/group/physicsgre_v2 April 15, 2009 Author: David S. Latchman

Upload: kev

Post on 01-Nov-2014

94 views

Category:

Documents


15 download

TRANSCRIPT

Page 1: 48578900 GRE Physics 8677 Solutions

DRAFT

The Physics GRE Solution Guide

GR8677 Test

http://groups.yahoo.com/group/physicsgre_v2

April 15, 2009

Author:David S. Latchman

Page 2: 48578900 GRE Physics 8677 Solutions

DRAFT

2

David S. Latchman ©2009

Page 3: 48578900 GRE Physics 8677 Solutions

DRAFTChapter 1

Preface

This solution guide initially started out on the Yahoo Groups web site and was prettysuccessful at the time. Unfortunately, the group was lost and with it, much of the thehard work that was put into it. This is my attempt to recreate the solution guide andmake it more widely avaialble to everyone. If you see any errors, think certain thingscould be expressed more clearly, or would like to make suggestions, please feel free todo so.

David Latchman

Document Changes

04-15-2009 First Version

Page 4: 48578900 GRE Physics 8677 Solutions

DRAFT

4 Preface

David S. Latchman ©2009

Page 5: 48578900 GRE Physics 8677 Solutions

DRAFTContents

1 Preface 3

2 Classical Mechanics 132.1 Kinematics . . . . . . . . . . . . . . . . . . . . . . . . . . . . . . . . . . . . 13

2.1.1 Linear Motion . . . . . . . . . . . . . . . . . . . . . . . . . . . . . . 132.1.2 Circular Motion . . . . . . . . . . . . . . . . . . . . . . . . . . . . . 13

2.2 Newton’s Laws . . . . . . . . . . . . . . . . . . . . . . . . . . . . . . . . . 142.2.1 Newton’s Laws of Motion . . . . . . . . . . . . . . . . . . . . . . . 142.2.2 Momentum . . . . . . . . . . . . . . . . . . . . . . . . . . . . . . . 142.2.3 Impulse . . . . . . . . . . . . . . . . . . . . . . . . . . . . . . . . . 14

2.3 Work & Energy . . . . . . . . . . . . . . . . . . . . . . . . . . . . . . . . . 152.3.1 Kinetic Energy . . . . . . . . . . . . . . . . . . . . . . . . . . . . . 152.3.2 The Work-Energy Theorem . . . . . . . . . . . . . . . . . . . . . . 152.3.3 Work done under a constant Force . . . . . . . . . . . . . . . . . . 152.3.4 Potential Energy . . . . . . . . . . . . . . . . . . . . . . . . . . . . 152.3.5 Hooke’s Law . . . . . . . . . . . . . . . . . . . . . . . . . . . . . . 152.3.6 Potential Energy of a Spring . . . . . . . . . . . . . . . . . . . . . . 15

2.4 Oscillatory Motion . . . . . . . . . . . . . . . . . . . . . . . . . . . . . . . 162.4.1 Equation for Simple Harmonic Motion . . . . . . . . . . . . . . . 162.4.2 Period of Simple Harmonic Motion . . . . . . . . . . . . . . . . . 162.4.3 Total Energy of an Oscillating System . . . . . . . . . . . . . . . . 162.4.4 Damped Harmonic Motion . . . . . . . . . . . . . . . . . . . . . . 162.4.5 Small Oscillations . . . . . . . . . . . . . . . . . . . . . . . . . . . . 172.4.6 Coupled Harmonic Oscillators . . . . . . . . . . . . . . . . . . . . 172.4.7 Doppler Effect . . . . . . . . . . . . . . . . . . . . . . . . . . . . . . 19

2.5 Rotational Motion about a Fixed Axis . . . . . . . . . . . . . . . . . . . . 202.5.1 Moment of Inertia . . . . . . . . . . . . . . . . . . . . . . . . . . . 202.5.2 Rotational Kinetic Energy . . . . . . . . . . . . . . . . . . . . . . . 202.5.3 Parallel Axis Theorem . . . . . . . . . . . . . . . . . . . . . . . . . 202.5.4 Torque . . . . . . . . . . . . . . . . . . . . . . . . . . . . . . . . . . 202.5.5 Angular Momentum . . . . . . . . . . . . . . . . . . . . . . . . . . 202.5.6 Kinetic Energy in Rolling . . . . . . . . . . . . . . . . . . . . . . . 21

2.6 Dynamics of Systems of Particles . . . . . . . . . . . . . . . . . . . . . . . 212.6.1 Center of Mass of a System of Particles . . . . . . . . . . . . . . . 21

2.7 Central Forces and Celestial Mechanics . . . . . . . . . . . . . . . . . . . 212.7.1 Newton’s Law of Universal Gravitation . . . . . . . . . . . . . . . 21

Page 6: 48578900 GRE Physics 8677 Solutions

DRAFT

6 Contents2.7.2 Potential Energy of a Gravitational Force . . . . . . . . . . . . . . 222.7.3 Escape Speed and Orbits . . . . . . . . . . . . . . . . . . . . . . . . 222.7.4 Kepler’s Laws . . . . . . . . . . . . . . . . . . . . . . . . . . . . . . 222.7.5 Types of Orbits . . . . . . . . . . . . . . . . . . . . . . . . . . . . . 222.7.6 Derivation of Vis-viva Equation . . . . . . . . . . . . . . . . . . . 23

2.8 Three Dimensional Particle Dynamics . . . . . . . . . . . . . . . . . . . . 232.9 Fluid Dynamics . . . . . . . . . . . . . . . . . . . . . . . . . . . . . . . . . 23

2.9.1 Archimedes’ Principle . . . . . . . . . . . . . . . . . . . . . . . . . 232.9.2 Equation of Continuity . . . . . . . . . . . . . . . . . . . . . . . . . 242.9.3 Bernoulli’s Equation . . . . . . . . . . . . . . . . . . . . . . . . . . 24

2.10 Non-inertial Reference Frames . . . . . . . . . . . . . . . . . . . . . . . . 242.11 Hamiltonian and Lagrangian Formalism . . . . . . . . . . . . . . . . . . . 24

2.11.1 Lagrange’s Function (L) . . . . . . . . . . . . . . . . . . . . . . . . 242.11.2 Equations of Motion(Euler-Lagrange Equation) . . . . . . . . . . 242.11.3 Hamiltonian . . . . . . . . . . . . . . . . . . . . . . . . . . . . . . . 24

3 Electromagnetism 253.1 Electrostatics . . . . . . . . . . . . . . . . . . . . . . . . . . . . . . . . . . . 25

3.1.1 Coulomb’s Law . . . . . . . . . . . . . . . . . . . . . . . . . . . . . 253.1.2 Electric Field of a point charge . . . . . . . . . . . . . . . . . . . . 253.1.3 Gauss’ Law . . . . . . . . . . . . . . . . . . . . . . . . . . . . . . . 263.1.4 Equivalence of Coulomb’s Law and Gauss’ Law . . . . . . . . . . 273.1.5 Electric Field due to a line of charge . . . . . . . . . . . . . . . . . 273.1.6 Electric Field in a Solid Non-Conducting Sphere . . . . . . . . . . 273.1.7 Electric Potential Energy . . . . . . . . . . . . . . . . . . . . . . . . 283.1.8 Electric Potential of a Point Charge . . . . . . . . . . . . . . . . . . 283.1.9 Electric Potential due to a line charge along axis . . . . . . . . . . 29

3.2 Currents and DC Circuits . . . . . . . . . . . . . . . . . . . . . . . . . . . 303.3 Magnetic Fields in Free Space . . . . . . . . . . . . . . . . . . . . . . . . . 303.4 Lorentz Force . . . . . . . . . . . . . . . . . . . . . . . . . . . . . . . . . . 303.5 Induction . . . . . . . . . . . . . . . . . . . . . . . . . . . . . . . . . . . . . 303.6 Maxwell’s Equations and their Applications . . . . . . . . . . . . . . . . . 303.7 Electromagnetic Waves . . . . . . . . . . . . . . . . . . . . . . . . . . . . . 303.8 AC Circuits . . . . . . . . . . . . . . . . . . . . . . . . . . . . . . . . . . . 303.9 Magnetic and Electric Fields in Matter . . . . . . . . . . . . . . . . . . . . 303.10 Capacitance . . . . . . . . . . . . . . . . . . . . . . . . . . . . . . . . . . . 303.11 Energy in a Capacitor . . . . . . . . . . . . . . . . . . . . . . . . . . . . . . 313.12 Energy in an Electric Field . . . . . . . . . . . . . . . . . . . . . . . . . . . 313.13 Current . . . . . . . . . . . . . . . . . . . . . . . . . . . . . . . . . . . . . . 313.14 Current Destiny . . . . . . . . . . . . . . . . . . . . . . . . . . . . . . . . . 313.15 Current Density of Moving Charges . . . . . . . . . . . . . . . . . . . . . 313.16 Resistance and Ohm’s Law . . . . . . . . . . . . . . . . . . . . . . . . . . 313.17 Resistivity and Conductivity . . . . . . . . . . . . . . . . . . . . . . . . . . 313.18 Power . . . . . . . . . . . . . . . . . . . . . . . . . . . . . . . . . . . . . . . 323.19 Kirchoff’s Loop Rules . . . . . . . . . . . . . . . . . . . . . . . . . . . . . . 323.20 Kirchoff’s Junction Rule . . . . . . . . . . . . . . . . . . . . . . . . . . . . 32

David S. Latchman ©2009

Page 7: 48578900 GRE Physics 8677 Solutions

DRAFT

Contents 73.21 RC Circuits . . . . . . . . . . . . . . . . . . . . . . . . . . . . . . . . . . . . 323.22 Maxwell’s Equations . . . . . . . . . . . . . . . . . . . . . . . . . . . . . . 32

3.22.1 Integral Form . . . . . . . . . . . . . . . . . . . . . . . . . . . . . . 323.22.2 Differential Form . . . . . . . . . . . . . . . . . . . . . . . . . . . . 33

3.23 Speed of Propagation of a Light Wave . . . . . . . . . . . . . . . . . . . . 333.24 Relationship between E and B Fields . . . . . . . . . . . . . . . . . . . . . 333.25 Energy Density of an EM wave . . . . . . . . . . . . . . . . . . . . . . . . 333.26 Poynting’s Vector . . . . . . . . . . . . . . . . . . . . . . . . . . . . . . . . 33

4 Optics & Wave Phonomena 354.1 Wave Properties . . . . . . . . . . . . . . . . . . . . . . . . . . . . . . . . . 354.2 Superposition . . . . . . . . . . . . . . . . . . . . . . . . . . . . . . . . . . 354.3 Interference . . . . . . . . . . . . . . . . . . . . . . . . . . . . . . . . . . . 354.4 Diffraction . . . . . . . . . . . . . . . . . . . . . . . . . . . . . . . . . . . . 354.5 Geometrical Optics . . . . . . . . . . . . . . . . . . . . . . . . . . . . . . . 354.6 Polarization . . . . . . . . . . . . . . . . . . . . . . . . . . . . . . . . . . . 354.7 Doppler Effect . . . . . . . . . . . . . . . . . . . . . . . . . . . . . . . . . . 354.8 Snell’s Law . . . . . . . . . . . . . . . . . . . . . . . . . . . . . . . . . . . . 36

4.8.1 Snell’s Law . . . . . . . . . . . . . . . . . . . . . . . . . . . . . . . 364.8.2 Critical Angle and Snell’s Law . . . . . . . . . . . . . . . . . . . . 36

5 Thermodynamics & Statistical Mechanics 375.1 Laws of Thermodynamics . . . . . . . . . . . . . . . . . . . . . . . . . . . 375.2 Thermodynamic Processes . . . . . . . . . . . . . . . . . . . . . . . . . . . 375.3 Equations of State . . . . . . . . . . . . . . . . . . . . . . . . . . . . . . . . 375.4 Ideal Gases . . . . . . . . . . . . . . . . . . . . . . . . . . . . . . . . . . . . 375.5 Kinetic Theory . . . . . . . . . . . . . . . . . . . . . . . . . . . . . . . . . . 375.6 Ensembles . . . . . . . . . . . . . . . . . . . . . . . . . . . . . . . . . . . . 375.7 Statistical Concepts and Calculation of Thermodynamic Properties . . . 385.8 Thermal Expansion & Heat Transfer . . . . . . . . . . . . . . . . . . . . . 385.9 Heat Capacity . . . . . . . . . . . . . . . . . . . . . . . . . . . . . . . . . . 385.10 Specific Heat Capacity . . . . . . . . . . . . . . . . . . . . . . . . . . . . . 385.11 Heat and Work . . . . . . . . . . . . . . . . . . . . . . . . . . . . . . . . . 385.12 First Law of Thermodynamics . . . . . . . . . . . . . . . . . . . . . . . . . 38

5.12.1 Special Cases to the First Law of Thermodynamics . . . . . . . . 385.13 Work done by Ideal Gas at Constant Temperature . . . . . . . . . . . . . 395.14 Heat Conduction Equation . . . . . . . . . . . . . . . . . . . . . . . . . . . 395.15 Ideal Gas Law . . . . . . . . . . . . . . . . . . . . . . . . . . . . . . . . . . 395.16 Stefan-Boltzmann’s Formula . . . . . . . . . . . . . . . . . . . . . . . . . . 405.17 RMS Speed of an Ideal Gas . . . . . . . . . . . . . . . . . . . . . . . . . . 405.18 Translational Kinetic Energy . . . . . . . . . . . . . . . . . . . . . . . . . . 405.19 Internal Energy of a Monatomic gas . . . . . . . . . . . . . . . . . . . . . 405.20 Molar Specific Heat at Constant Volume . . . . . . . . . . . . . . . . . . . 405.21 Molar Specific Heat at Constant Pressure . . . . . . . . . . . . . . . . . . 415.22 Equipartition of Energy . . . . . . . . . . . . . . . . . . . . . . . . . . . . 415.23 Adiabatic Expansion of an Ideal Gas . . . . . . . . . . . . . . . . . . . . . 41

©2009 David S. Latchman

Page 8: 48578900 GRE Physics 8677 Solutions

DRAFT

8 Contents5.24 Second Law of Thermodynamics . . . . . . . . . . . . . . . . . . . . . . . 41

6 Quantum Mechanics 436.1 Fundamental Concepts . . . . . . . . . . . . . . . . . . . . . . . . . . . . . 436.2 Schrodinger Equation . . . . . . . . . . . . . . . . . . . . . . . . . . . . . . 43

6.2.1 Infinite Square Wells . . . . . . . . . . . . . . . . . . . . . . . . . . 446.2.2 Harmonic Oscillators . . . . . . . . . . . . . . . . . . . . . . . . . . 456.2.3 Finite Square Well . . . . . . . . . . . . . . . . . . . . . . . . . . . 476.2.4 Hydrogenic Atoms . . . . . . . . . . . . . . . . . . . . . . . . . . . 48

6.3 Spin . . . . . . . . . . . . . . . . . . . . . . . . . . . . . . . . . . . . . . . . 486.4 Angular Momentum . . . . . . . . . . . . . . . . . . . . . . . . . . . . . . 486.5 Wave Funtion Symmetry . . . . . . . . . . . . . . . . . . . . . . . . . . . . 486.6 Elementary Perturbation Theory . . . . . . . . . . . . . . . . . . . . . . . 48

7 Atomic Physics 497.1 Properties of Electrons . . . . . . . . . . . . . . . . . . . . . . . . . . . . . 497.2 Bohr Model . . . . . . . . . . . . . . . . . . . . . . . . . . . . . . . . . . . 497.3 Energy Quantization . . . . . . . . . . . . . . . . . . . . . . . . . . . . . . 507.4 Atomic Structure . . . . . . . . . . . . . . . . . . . . . . . . . . . . . . . . 507.5 Atomic Spectra . . . . . . . . . . . . . . . . . . . . . . . . . . . . . . . . . 51

7.5.1 Rydberg’s Equation . . . . . . . . . . . . . . . . . . . . . . . . . . . 517.6 Selection Rules . . . . . . . . . . . . . . . . . . . . . . . . . . . . . . . . . . 517.7 Black Body Radiation . . . . . . . . . . . . . . . . . . . . . . . . . . . . . . 51

7.7.1 Plank Formula . . . . . . . . . . . . . . . . . . . . . . . . . . . . . 517.7.2 Stefan-Boltzmann Formula . . . . . . . . . . . . . . . . . . . . . . 517.7.3 Wein’s Displacement Law . . . . . . . . . . . . . . . . . . . . . . . 517.7.4 Classical and Quantum Aspects of the Plank Equation . . . . . . 51

7.8 X-Rays . . . . . . . . . . . . . . . . . . . . . . . . . . . . . . . . . . . . . . 527.8.1 Bragg Condition . . . . . . . . . . . . . . . . . . . . . . . . . . . . 527.8.2 The Compton Effect . . . . . . . . . . . . . . . . . . . . . . . . . . 52

7.9 Atoms in Electric and Magnetic Fields . . . . . . . . . . . . . . . . . . . . 537.9.1 The Cyclotron Frequency . . . . . . . . . . . . . . . . . . . . . . . 537.9.2 Zeeman Effect . . . . . . . . . . . . . . . . . . . . . . . . . . . . . . 547.9.3 Franck-Hertz Experiment . . . . . . . . . . . . . . . . . . . . . . . 55

8 Special Relativity 578.1 Introductory Concepts . . . . . . . . . . . . . . . . . . . . . . . . . . . . . 57

8.1.1 Postulates of Special Relativity . . . . . . . . . . . . . . . . . . . . 578.2 Time Dilation . . . . . . . . . . . . . . . . . . . . . . . . . . . . . . . . . . 578.3 Length Contraction . . . . . . . . . . . . . . . . . . . . . . . . . . . . . . . 578.4 Simultaneity . . . . . . . . . . . . . . . . . . . . . . . . . . . . . . . . . . . 578.5 Energy and Momentum . . . . . . . . . . . . . . . . . . . . . . . . . . . . 58

8.5.1 Relativistic Momentum & Energy . . . . . . . . . . . . . . . . . . 588.5.2 Lorentz Transformations (Momentum & Energy) . . . . . . . . . 588.5.3 Relativistic Kinetic Energy . . . . . . . . . . . . . . . . . . . . . . . 588.5.4 Relativistic Dynamics (Collisions) . . . . . . . . . . . . . . . . . . 58

David S. Latchman ©2009

Page 9: 48578900 GRE Physics 8677 Solutions

DRAFT

Contents 98.6 Four-Vectors and Lorentz Transformation . . . . . . . . . . . . . . . . . . 598.7 Velocity Addition . . . . . . . . . . . . . . . . . . . . . . . . . . . . . . . . 608.8 Relativistic Doppler Formula . . . . . . . . . . . . . . . . . . . . . . . . . 608.9 Lorentz Transformations . . . . . . . . . . . . . . . . . . . . . . . . . . . . 608.10 Space-Time Interval . . . . . . . . . . . . . . . . . . . . . . . . . . . . . . . 61

9 Laboratory Methods 639.1 Data and Error Analysis . . . . . . . . . . . . . . . . . . . . . . . . . . . . 63

9.1.1 Addition and Subtraction . . . . . . . . . . . . . . . . . . . . . . . 639.1.2 Multiplication and Division . . . . . . . . . . . . . . . . . . . . . . 639.1.3 Exponent - (No Error in b) . . . . . . . . . . . . . . . . . . . . . . . 639.1.4 Logarithms . . . . . . . . . . . . . . . . . . . . . . . . . . . . . . . 639.1.5 Antilogs . . . . . . . . . . . . . . . . . . . . . . . . . . . . . . . . . 64

9.2 Instrumentation . . . . . . . . . . . . . . . . . . . . . . . . . . . . . . . . . 659.3 Radiation Detection . . . . . . . . . . . . . . . . . . . . . . . . . . . . . . . 659.4 Counting Statistics . . . . . . . . . . . . . . . . . . . . . . . . . . . . . . . 659.5 Interaction of Charged Particles with Matter . . . . . . . . . . . . . . . . 659.6 Lasers and Optical Interferometers . . . . . . . . . . . . . . . . . . . . . . 659.7 Dimensional Analysis . . . . . . . . . . . . . . . . . . . . . . . . . . . . . 669.8 Fundamental Applications of Probability and Statistics . . . . . . . . . . 66

10 GR8677 Exam Solutions 6710.1 Motion of Rock under Drag Force . . . . . . . . . . . . . . . . . . . . . . . 6710.2 Satellite Orbits . . . . . . . . . . . . . . . . . . . . . . . . . . . . . . . . . . 6810.3 Speed of Light in a Dielectric Medium . . . . . . . . . . . . . . . . . . . . 6810.4 Wave Equation . . . . . . . . . . . . . . . . . . . . . . . . . . . . . . . . . 6810.5 Inelastic Collision and Putty Spheres . . . . . . . . . . . . . . . . . . . . . 6910.6 Motion of a Particle along a Track . . . . . . . . . . . . . . . . . . . . . . . 7010.7 Resolving Force Components . . . . . . . . . . . . . . . . . . . . . . . . . 7010.8 Nail being driven into a block of wood . . . . . . . . . . . . . . . . . . . . 7110.9 Current Density . . . . . . . . . . . . . . . . . . . . . . . . . . . . . . . . . 7110.10Charge inside an Isolated Sphere . . . . . . . . . . . . . . . . . . . . . . . 7210.11Vector Identities and Maxwell’s Laws . . . . . . . . . . . . . . . . . . . . 7310.12Doppler Equation (Non-Relativistic) . . . . . . . . . . . . . . . . . . . . . 7310.13Vibrating Interference Pattern . . . . . . . . . . . . . . . . . . . . . . . . . 7310.14Specific Heat at Constant Pressure and Volume . . . . . . . . . . . . . . . 7310.15Helium atoms in a box . . . . . . . . . . . . . . . . . . . . . . . . . . . . . 7410.16The Muon . . . . . . . . . . . . . . . . . . . . . . . . . . . . . . . . . . . . 7510.17Radioactive Decay . . . . . . . . . . . . . . . . . . . . . . . . . . . . . . . 7510.18Schrodinger’s Equation . . . . . . . . . . . . . . . . . . . . . . . . . . . . 7510.19Energy Levels of Bohr’s Hydrogen Atom . . . . . . . . . . . . . . . . . . 7610.20Relativistic Energy . . . . . . . . . . . . . . . . . . . . . . . . . . . . . . . 7610.21Space-Time Interval . . . . . . . . . . . . . . . . . . . . . . . . . . . . . . . 7710.22Lorentz Transformation of the EM field . . . . . . . . . . . . . . . . . . . 7710.23Conductivity of a Metal and Semi-Conductor . . . . . . . . . . . . . . . . 7810.24Charging a Battery . . . . . . . . . . . . . . . . . . . . . . . . . . . . . . . 78

©2009 David S. Latchman

Page 10: 48578900 GRE Physics 8677 Solutions

DRAFT

10 Contents10.25Lorentz Force on a Charged Particle . . . . . . . . . . . . . . . . . . . . . 7810.26K-Series X-Rays . . . . . . . . . . . . . . . . . . . . . . . . . . . . . . . . . 7910.27Electrons and Spin . . . . . . . . . . . . . . . . . . . . . . . . . . . . . . . 7910.28Normalizing a wavefunction . . . . . . . . . . . . . . . . . . . . . . . . . 8110.29Right Hand Rule . . . . . . . . . . . . . . . . . . . . . . . . . . . . . . . . 8110.30Electron Configuration of a Potassium atom . . . . . . . . . . . . . . . . . 8110.31Photoelectric Effect I . . . . . . . . . . . . . . . . . . . . . . . . . . . . . . 8210.32Photoelectric Effect II . . . . . . . . . . . . . . . . . . . . . . . . . . . . . . 8210.33Photoelectric Effect III . . . . . . . . . . . . . . . . . . . . . . . . . . . . . 8210.34Potential Energy of a Body . . . . . . . . . . . . . . . . . . . . . . . . . . . 8210.35Hamiltonian of a Body . . . . . . . . . . . . . . . . . . . . . . . . . . . . . 8310.36Principle of Least Action . . . . . . . . . . . . . . . . . . . . . . . . . . . . 8310.37Tension in a Conical Pendulum . . . . . . . . . . . . . . . . . . . . . . . . 8310.38Diode OR-gate . . . . . . . . . . . . . . . . . . . . . . . . . . . . . . . . . . 8410.39Gain of an Amplifier vs. Angular Frequency . . . . . . . . . . . . . . . . 8410.40Counting Statistics . . . . . . . . . . . . . . . . . . . . . . . . . . . . . . . 8410.41Binding Energy per Nucleon . . . . . . . . . . . . . . . . . . . . . . . . . . 8510.42Scattering Cross Section . . . . . . . . . . . . . . . . . . . . . . . . . . . . 8510.43Coupled Oscillators . . . . . . . . . . . . . . . . . . . . . . . . . . . . . . . 85

10.43.1 Calculating the modes of oscillation . . . . . . . . . . . . . . . . . 8510.44Collision with a Rod . . . . . . . . . . . . . . . . . . . . . . . . . . . . . . 8610.45Compton Wavelength . . . . . . . . . . . . . . . . . . . . . . . . . . . . . 8710.46Stefan-Boltzmann’s Equation . . . . . . . . . . . . . . . . . . . . . . . . . 8710.47Franck-Hertz Experiment . . . . . . . . . . . . . . . . . . . . . . . . . . . 8710.48Selection Rules for Electronic Transitions . . . . . . . . . . . . . . . . . . 8710.49The Hamilton Operator . . . . . . . . . . . . . . . . . . . . . . . . . . . . 8810.50Hall Effect . . . . . . . . . . . . . . . . . . . . . . . . . . . . . . . . . . . . 8810.51Debye and Einstein Theories to Specific Heat . . . . . . . . . . . . . . . . 8910.52Potential inside a Hollow Cube . . . . . . . . . . . . . . . . . . . . . . . . 9010.53EM Radiation from Oscillating Charges . . . . . . . . . . . . . . . . . . . 9010.54Polarization Charge Density . . . . . . . . . . . . . . . . . . . . . . . . . . 9010.55Kinetic Energy of Electrons in Metals . . . . . . . . . . . . . . . . . . . . . 9110.56Expectation or Mean Value . . . . . . . . . . . . . . . . . . . . . . . . . . . 9110.57Eigenfuction of Wavefunction . . . . . . . . . . . . . . . . . . . . . . . . . 9110.58Holograms . . . . . . . . . . . . . . . . . . . . . . . . . . . . . . . . . . . . 9210.59Group Velocity of a Wave . . . . . . . . . . . . . . . . . . . . . . . . . . . 9310.60Potential Energy and Simple Harmonic Motion . . . . . . . . . . . . . . . 9310.61Rocket Equation I . . . . . . . . . . . . . . . . . . . . . . . . . . . . . . . . 9410.62Rocket Equation II . . . . . . . . . . . . . . . . . . . . . . . . . . . . . . . 9410.63Surface Charge Density . . . . . . . . . . . . . . . . . . . . . . . . . . . . . 9410.64Maximum Power Theorem . . . . . . . . . . . . . . . . . . . . . . . . . . . 9510.65Magnetic Field far away from a Current carrying Loop . . . . . . . . . . 9510.66Maxwell’s Relations . . . . . . . . . . . . . . . . . . . . . . . . . . . . . . . 9610.67Partition Functions . . . . . . . . . . . . . . . . . . . . . . . . . . . . . . . 9710.68Particle moving at Light Speed . . . . . . . . . . . . . . . . . . . . . . . . 9710.69Car and Garage I . . . . . . . . . . . . . . . . . . . . . . . . . . . . . . . . 97

David S. Latchman ©2009

Page 11: 48578900 GRE Physics 8677 Solutions

DRAFT

Contents 1110.70Car and Garage II . . . . . . . . . . . . . . . . . . . . . . . . . . . . . . . . 9710.71Car and Garage III . . . . . . . . . . . . . . . . . . . . . . . . . . . . . . . 9810.72Refrective Index of Rock Salt and X-rays . . . . . . . . . . . . . . . . . . . 9810.73Thin Flim Non-Reflective Coatings . . . . . . . . . . . . . . . . . . . . . . 9910.74Law of Malus . . . . . . . . . . . . . . . . . . . . . . . . . . . . . . . . . . 9910.75Geosynchronous Satellite Orbit . . . . . . . . . . . . . . . . . . . . . . . . 10010.76Hoop Rolling down and Inclined Plane . . . . . . . . . . . . . . . . . . . 10010.77Simple Harmonic Motion . . . . . . . . . . . . . . . . . . . . . . . . . . . 10110.78Total Energy between Two Charges . . . . . . . . . . . . . . . . . . . . . . 10110.79Maxwell’s Equations and Magnetic Monopoles . . . . . . . . . . . . . . . 10210.80Gauss’ Law . . . . . . . . . . . . . . . . . . . . . . . . . . . . . . . . . . . . 10210.81Biot-Savart Law . . . . . . . . . . . . . . . . . . . . . . . . . . . . . . . . . 10310.82Zeeman Effect and the emission spectrum of atomic gases . . . . . . . . 10410.83Spectral Lines in High Density and Low Density Gases . . . . . . . . . . 10510.84Term Symbols & Spectroscopic Notation . . . . . . . . . . . . . . . . . . . 10510.85Photon Interaction Cross Sections for Pb . . . . . . . . . . . . . . . . . . . 10610.86The Ice Pail Experiment . . . . . . . . . . . . . . . . . . . . . . . . . . . . 10610.87Equipartition of Energy and Diatomic Molecules . . . . . . . . . . . . . . 10610.88Fermion and Boson Pressure . . . . . . . . . . . . . . . . . . . . . . . . . . 10710.89Wavefunction of Two Identical Particles . . . . . . . . . . . . . . . . . . . 10710.90Energy Eigenstates . . . . . . . . . . . . . . . . . . . . . . . . . . . . . . . 10810.91Bragg’s Law . . . . . . . . . . . . . . . . . . . . . . . . . . . . . . . . . . . 10810.92Selection Rules for Electronic Transitions . . . . . . . . . . . . . . . . . . 10910.93Moving Belt Sander on a Rough Plane . . . . . . . . . . . . . . . . . . . . 10910.94RL Circuits . . . . . . . . . . . . . . . . . . . . . . . . . . . . . . . . . . . . 11010.95Carnot Cycles . . . . . . . . . . . . . . . . . . . . . . . . . . . . . . . . . . 11110.96First Order Perturbation Theory . . . . . . . . . . . . . . . . . . . . . . . . 11310.97Colliding Discs and the Conservation of Angular Momentum . . . . . . 11410.98Electrical Potential of a Long Thin Rod . . . . . . . . . . . . . . . . . . . . 11410.99Ground State of a Positronium Atom . . . . . . . . . . . . . . . . . . . . . 11510.100The Pinhole Camera . . . . . . . . . . . . . . . . . . . . . . . . . . . . . . 115

A Constants & Important Equations 117A.1 Constants . . . . . . . . . . . . . . . . . . . . . . . . . . . . . . . . . . . . . 117A.2 Vector Identities . . . . . . . . . . . . . . . . . . . . . . . . . . . . . . . . . 117

A.2.1 Triple Products . . . . . . . . . . . . . . . . . . . . . . . . . . . . . 117A.2.2 Product Rules . . . . . . . . . . . . . . . . . . . . . . . . . . . . . . 118A.2.3 Second Derivatives . . . . . . . . . . . . . . . . . . . . . . . . . . . 118

A.3 Commutators . . . . . . . . . . . . . . . . . . . . . . . . . . . . . . . . . . 118A.3.1 Lie-algebra Relations . . . . . . . . . . . . . . . . . . . . . . . . . . 118A.3.2 Canonical Commutator . . . . . . . . . . . . . . . . . . . . . . . . 118A.3.3 Kronecker Delta Function . . . . . . . . . . . . . . . . . . . . . . . 118

A.4 Linear Algebra . . . . . . . . . . . . . . . . . . . . . . . . . . . . . . . . . . 118A.4.1 Vectors . . . . . . . . . . . . . . . . . . . . . . . . . . . . . . . . . . 118

©2009 David S. Latchman

Page 12: 48578900 GRE Physics 8677 Solutions

DRAFT

12 Contents

David S. Latchman ©2009

Page 13: 48578900 GRE Physics 8677 Solutions

DRAFTChapter 2

Classical Mechanics

2.1 Kinematics

2.1.1 Linear Motion

Average Velocity

v =∆x∆t

=x2 − x1

t2 − t1(2.1)

Instantaneous Velocity

v = lim∆t→0

∆x∆t

=dxdt

= v(t) (2.2)

Kinematic Equations of Motion

The basic kinematic equations of motion under constant acceleration, a, are

v = v0 + at (2.3)

v2 = v20 + 2a (x − x0) (2.4)

x − x0 = v0t +12

at2 (2.5)

x − x0 =12

(v + v0) t (2.6)

2.1.2 Circular Motion

In the case of Uniform Circular Motion, for a particle to move in a circular path, aradial acceleration must be applied. This acceleration is known as the CentripetalAcceleration

Centripetal Acceleration

a =v2

r(2.7)

Page 14: 48578900 GRE Physics 8677 Solutions

DRAFT

14 Classical MechanicsAngular Velocity

ω =vr

(2.8)

We can write (Equation 2.7) in terms of ω

a = ω2r (2.9)

Rotational Equations of Motion

The equations of motion under a constant angular acceleration, α, are

ω = ω0 + αt (2.10)

θ =ω + ω0

2t (2.11)

θ = ω0t +12αt2 (2.12)

ω2 = ω20 + 2αθ (2.13)

2.2 Newton’s Laws

2.2.1 Newton’s Laws of Motion

First Law A body continues in its state of rest or of uniform motion unless acted uponby an external unbalanced force.

Second Law The net force on a body is proportional to its rate of change of momentum.

F =dpdt

= ma (2.14)

Third Law When a particle A exerts a force on another particle B, B simultaneouslyexerts a force on A with the same magnitude in the opposite direction.

FAB = −FBA (2.15)

2.2.2 Momentum

p = mv (2.16)

2.2.3 Impulse

∆p = J =w

Fdt = Favgdt (2.17)

David S. Latchman ©2009

Page 15: 48578900 GRE Physics 8677 Solutions

DRAFT

Work & Energy 152.3 Work & Energy

2.3.1 Kinetic Energy

K ≡12

mv2 (2.18)

2.3.2 The Work-Energy Theorem

The net Work done is given byWnet = K f − Ki (2.19)

2.3.3 Work done under a constant Force

The work done by a force can be expressed as

W = F∆x (2.20)

In three dimensions, this becomes

W = F · ∆r = F∆r cosθ (2.21)

For a non-constant force, we have

W =

x fw

xi

F(x)dx (2.22)

2.3.4 Potential Energy

The Potential Energy is

F(x) = −dU(x)

dx(2.23)

for conservative forces, the potential energy is

U(x) = U0 −

xw

x0

F(x′)dx′ (2.24)

2.3.5 Hooke’s Law

F = −kx (2.25)

where k is the spring constant.

2.3.6 Potential Energy of a Spring

U(x) =12

kx2 (2.26)

©2009 David S. Latchman

Page 16: 48578900 GRE Physics 8677 Solutions

DRAFT

16 Classical Mechanics2.4 Oscillatory Motion

2.4.1 Equation for Simple Harmonic Motion

x(t) = A sin (ωt + δ) (2.27)

where the Amplitude, A, measures the displacement from equilibrium, the phase, δ, isthe angle by which the motion is shifted from equilibrium at t = 0.

2.4.2 Period of Simple Harmonic Motion

T =2πω

(2.28)

2.4.3 Total Energy of an Oscillating System

Given thatx = A sin (ωt + δ) (2.29)

and that the Total Energy of a System is

E = KE + PE (2.30)

The Kinetic Energy is

KE =12

mv2

=12

mdxdt

=12

mA2ω2 cos2 (ωt + δ) (2.31)

The Potential Energy is

U =12

kx2

=12

kA2 sin2 (ωt + δ) (2.32)

Adding (Equation 2.31) and (Equation 2.32) gives

E =12

kA2 (2.33)

2.4.4 Damped Harmonic Motion

Fd = −bv = −bdxdt

(2.34)

where b is the damping coefficient. The equation of motion for a damped oscillatingsystem becomes

− kx − bdxdt

= md2xdt2 (2.35)

David S. Latchman ©2009

Page 17: 48578900 GRE Physics 8677 Solutions

DRAFT

Oscillatory Motion 17Solving(Equation 2.35) goves

x = Ae−αt sin (ω′t + δ) (2.36)

We find that

α =b

2m(2.37)

ω′ =

√km−

b2

4m2

=

√ω2

0 −b2

4m2

=√ω2

0 − α2 (2.38)

2.4.5 Small Oscillations

The Energy of a system is

E = K + V(x) =12

mv(x)2 + V(x) (2.39)

We can solve for v(x),

v(x) =

√2m

(E − V(x)) (2.40)

where E ≥ V(x) Let the particle move in the potential valley, x1 ≤ x ≤ x2, the potentialcan be approximated by the Taylor Expansion

V(x) = V(xe) + (x − xe)[dV(x)

dx

]x=xe

+12

(x − xe)2

[d2V(x)

dx2

]x=xe

+ · · · (2.41)

At the points of inflection, the derivative dV/dx is zero and d2V/dx2 is positive. Thismeans that the potential energy for small oscillations becomes

V(x) u V(xe) +12

k(x − xe)2 (2.42)

where

k ≡[d2V(x)

dx2

]x=xe

≥ 0 (2.43)

As V(xe) is constant, it has no consequences to physical motion and can be dropped.We see that Equation 2.42 is that of simple harmonic motion.

2.4.6 Coupled Harmonic Oscillators

Consider the case of a simple pendulum of length, `, and the mass of the bob is m1.For small displacements, the equation of motion is

θ + ω0θ = 0 (2.44)1Add figure with coupled pendulum-spring system

©2009 David S. Latchman

Page 18: 48578900 GRE Physics 8677 Solutions

DRAFT

18 Classical MechanicsWe can express this in cartesian coordinates, x and y, where

x = ` cosθ ≈ ` (2.45)y = ` sinθ ≈ `θ (2.46)

Equation 2.44 becomesy + ω0y = 0 (2.47)

This is the equivalent to the mass-spring system where the spring constant is

k = mω20 =

mg`

(2.48)

This allows us to to create an equivalent three spring system to our coupled pendulumsystem. The equations of motion can be derived from the Lagrangian, where

L = T − V

=12

my21 +

12

my22 −

(12

ky21 +

12κ(y2 − y1

)2+

12

ky22

)=

12

m(y1

2 + y22)−

12

(k(y2

1 + y22

)+ κ

(y2 − y1

)2)

(2.49)

We can find the equations of motion of our system

ddt

(∂L∂yn

)=∂L∂yn

(2.50)

The equations of motion are

my1 = −ky1 + κ(y2 − y1

)(2.51)

my2 = −ky2 + κ(y2 − y1

)(2.52)

We assume solutions for the equations of motion to be of the form

y1 = cos(ωt + δ1) y2 = B cos(ωt + δ2)y1 = −ωy1 y2 = −ωy2

(2.53)

Substituting the values for y1 and y2 into the equations of motion yields(k + κ −mω2

)y1 − κy2 = 0 (2.54)

−κy1 +(k + κ −mω2

)y2 = 0 (2.55)

We can get solutions from solving the determinant of the matrix∣∣∣∣∣(k + κ −mω2)−κ

−κ(k + κ −mω2)∣∣∣∣∣ = 0 (2.56)

Solving the determinant gives(mω2

)2− 2mω2 (k + κ) +

(k2 + 2kκ

)= 0 (2.57)

David S. Latchman ©2009

Page 19: 48578900 GRE Physics 8677 Solutions

DRAFT

Oscillatory Motion 19This yields

ω2 =

km

=g`

k + 2κm

=g`

+2κm

(2.58)

We can now determine exactly how the masses move with each mode by substitutingω2 into the equations of motion. Where

ω2 =km

We see that

k + κ −mω2 = κ (2.59)

Substituting this into the equation of motion yields

y1 = y2 (2.60)

We see that the masses move in phase with each other. You will also noticethe absense of the spring constant term, κ, for the connecting spring. As themasses are moving in step, the spring isn’t stretching or compressing and henceits absence in our result.

ω2 =k + κ

mWe see that

k + κ −mω2 = −κ (2.61)

Substituting this into the equation of motion yields

y1 = −y2 (2.62)

Here the masses move out of phase with each other. In this case we see thepresence of the spring constant, κ, which is expected as the spring playes a role.It is being stretched and compressed as our masses oscillate.

2.4.7 Doppler Effect

The Doppler Effect is the shift in frequency and wavelength of waves that results froma source moving with respect to the medium, a receiver moving with respect to themedium or a moving medium.

Moving Source If a source is moving towards an observer, then in one period, τ0, itmoves a distance of vsτ0 = vs/ f0. The wavelength is decreased by

λ′ = λ −vs

f0−

v − vs

f0(2.63)

The frequency change is

f ′ =vλ′

= f0

( vv − vs

)(2.64)

©2009 David S. Latchman

Page 20: 48578900 GRE Physics 8677 Solutions

DRAFT

20 Classical MechanicsMoving Observer As the observer moves, he will measure the same wavelength, λ, as

if at rest but will see the wave crests pass by more quickly. The observer measuresa modified wave speed.

v′ = v + |vr| (2.65)

The modified frequency becomes

f ′ =v′

λ= f0

(1 +

vr

v

)(2.66)

Moving Source and Moving Observer We can combine the above two equations

λ′ =v − vs

f0(2.67)

v′ = v − vr (2.68)

To give a modified frequency of

f ′ =v′

λ′=

(v − vr

v − vs

)f0 (2.69)

2.5 Rotational Motion about a Fixed Axis

2.5.1 Moment of Inertia

I =

∫R2dm (2.70)

2.5.2 Rotational Kinetic Energy

K =12

Iω2 (2.71)

2.5.3 Parallel Axis Theorem

I = Icm + Md2 (2.72)

2.5.4 Torque

τ = r × F (2.73)τ = Iα (2.74)

where α is the angular acceleration.

2.5.5 Angular Momentum

L = Iω (2.75)

we can find the Torque

τ =dLdt

(2.76)

David S. Latchman ©2009

Page 21: 48578900 GRE Physics 8677 Solutions

DRAFT

Dynamics of Systems of Particles 212.5.6 Kinetic Energy in Rolling

With respect to the point of contact, the motion of the wheel is a rotation about thepoint of contact. Thus

K = Krot =12

Icontactω2 (2.77)

Icontact can be found from the Parallel Axis Theorem.

Icontact = Icm + MR2 (2.78)

Substitute (Equation 2.77) and we have

K =12

(Icm + MR2

)ω2

=12

Icmω2 +

12

mv2 (2.79)

The kinetic energy of an object rolling without slipping is the sum of hte kinetic energyof rotation about its center of mass and the kinetic energy of the linear motion of theobject.

2.6 Dynamics of Systems of Particles

2.6.1 Center of Mass of a System of Particles

Position Vector of a System of Particles

R =m1r1 + m2r2 + m3r3 + · · · + mNrN

M(2.80)

Velocity Vector of a System of Particles

V =dRdt

=m1v1 + m2v2 + m3v3 + · · · + mNvN

M(2.81)

Acceleration Vector of a System of Particles

A =dVdt

=m1a1 + m2a2 + m3a3 + · · · + mNaN

M(2.82)

2.7 Central Forces and Celestial Mechanics

2.7.1 Newton’s Law of Universal Gravitation

F = −(GMm

r2

)r (2.83)

©2009 David S. Latchman

Page 22: 48578900 GRE Physics 8677 Solutions

DRAFT

22 Classical Mechanics2.7.2 Potential Energy of a Gravitational Force

U(r) = −GMm

r(2.84)

2.7.3 Escape Speed and Orbits

The energy of an orbiting body is

E = T + U

=12

mv2−

GMmr

(2.85)

The escape speed becomes

E =12

mv2esc −

GMmRE

= 0 (2.86)

Solving for vesc we find

vesc =

√2GM

Re(2.87)

2.7.4 Kepler’s Laws

First Law The orbit of every planet is an ellipse with the sun at a focus.

Second Law A line joining a planet and the sun sweeps out equal areas during equalintervals of time.

Third Law The square of the orbital period of a planet is directly proportional to thecube of the semi-major axis of its orbit.

T2

R3 = C (2.88)

where C is a constant whose value is the same for all planets.

2.7.5 Types of Orbits

The Energy of an Orbiting Body is defined in (Equation 2.85), we can classify orbits bytheir eccentricities.

Circular Orbit A circular orbit occurs when there is an eccentricity of 0 and the orbitalenergy is less than 0. Thus

12

v2−

GMr

= E < 0 (2.89)

The Orbital Velocity is

v =

√GM

r(2.90)

David S. Latchman ©2009

Page 23: 48578900 GRE Physics 8677 Solutions

DRAFT

Three Dimensional Particle Dynamics 23Elliptic Orbit An elliptic orbit occurs when the eccentricity is between 0 and 1 but the

specific energy is negative, so the object remains bound.

v =

√GM

(2r−

1a

)(2.91)

where a is the semi-major axis

Parabolic Orbit A Parabolic Orbit occurs when the eccentricity is equal to 1 and theorbital velocity is the escape velocity. This orbit is not bounded. Thus

12

v2−

GMr

= E = 0 (2.92)

The Orbital Velocity is

v = vesc =

√2GM

r(2.93)

Hyperbolic Orbit In the Hyperbolic Orbit, the eccentricity is greater than 1 with anorbital velocity in excess of the escape velocity. This orbit is also not bounded.

v∞ =

√GM

a(2.94)

2.7.6 Derivation of Vis-viva Equation

The total energy of a satellite is

E =12

mv2−

GMmr

(2.95)

For an elliptical or circular orbit, the specific energy is

E = −GMm

2a(2.96)

Equating we get

v2 = GM(2

r−

1a

)(2.97)

2.8 Three Dimensional Particle Dynamics

2.9 Fluid Dynamics

2.9.1 Archimedes’ Principle

When an object is fully or partially immersed, the buoyant force is equal to the weightof fluid displaced.

©2009 David S. Latchman

Page 24: 48578900 GRE Physics 8677 Solutions

DRAFT

24 Classical Mechanics2.9.2 Equation of Continuity

ρ1v1A1 = ρ2v2A2 (2.98)

2.9.3 Bernoulli’s Equation

P +12ρv2 + ρgh = a constant (2.99)

2.10 Non-inertial Reference Frames

2.11 Hamiltonian and Lagrangian Formalism

2.11.1 Lagrange’s Function (L)

L = T − V (2.100)

where T is the Kinetic Energy and V is the Potential Energy in terms of GeneralizedCoordinates.

2.11.2 Equations of Motion(Euler-Lagrange Equation)

∂L∂q

=ddt

(∂L∂q

)(2.101)

2.11.3 Hamiltonian

H = T + V= pq − L(q, q) (2.102)

where

∂H∂p

= q (2.103)

∂H∂q

= −∂L∂x

= −p (2.104)

David S. Latchman ©2009

Page 25: 48578900 GRE Physics 8677 Solutions

DRAFTChapter 3

Electromagnetism

3.1 Electrostatics

3.1.1 Coulomb’s Law

The force between two charged particles, q1 and q2 is defined by Coulomb’s Law.

F12 =1

4πε0

(q1q2

r212

)r12 (3.1)

where ε0 is the permitivitty of free space, where

ε0 = 8.85 × 10−12C2N.m2 (3.2)

3.1.2 Electric Field of a point charge

The electric field is defined by mesuring the magnitide and direction of an electricforce, F, acting on a test charge, q0.

E ≡Fq0

(3.3)

The Electric Field of a point charge, q is

E =1

4πε0

qr2 r (3.4)

In the case of multiple point charges, qi, the electric field becomes

E(r) =1

4πε0

n∑i=1

qi

r2i

ri (3.5)

Page 26: 48578900 GRE Physics 8677 Solutions

DRAFT

26 ElectromagnetismElectric Fields and Continuous Charge Distributions

If a source is distributed continuously along a region of space, Equation 3.5 becomes

E(r) =1

4πε0

∫1r2 rdq (3.6)

If the charge was distributed along a line with linear charge density, λ,

λ =dqdx

(3.7)

The Electric Field of a line charge becomes

E(r) =1

4πε0

∫line

λr2 rdx (3.8)

In the case where the charge is distributed along a surface, the surface charge densityis, σ

σ =QA

=dqdA

(3.9)

The electric field along the surface becomes

E(r) =1

4πε0

∫Surface

σr2 rdA (3.10)

In the case where the charge is distributed throughout a volume, V, the volume chargedensity is

ρ =QV

=dqdV

(3.11)

The Electric Field is

E(r) =1

4πε0

∫Volume

ρ

r2 rdV (3.12)

3.1.3 Gauss’ Law

The electric field through a surface is

Φ =

∮surface S

dΦ =

∮surface S

E · dA (3.13)

The electric flux through a closed surface encloses a net charge.∮E · dA =

Qε0

(3.14)

where Q is the charge enclosed by our surface.

David S. Latchman ©2009

Page 27: 48578900 GRE Physics 8677 Solutions

DRAFT

Electrostatics 273.1.4 Equivalence of Coulomb’s Law and Gauss’ Law

The total flux through a sphere is∮E · dA = E(4πr2) =

qε0

(3.15)

From the above, we see that the electric field is

E =q

4πε0r2 (3.16)

3.1.5 Electric Field due to a line of charge

Consider an infinite rod of constant charge density, λ. The flux through a Gaussiancylinder enclosing the line of charge is

Φ =

∫top surface

E · dA +

∫bottom surface

E · dA +

∫side surface

E · dA (3.17)

At the top and bottom surfaces, the electric field is perpendicular to the area vector, sofor the top and bottom surfaces,

E · dA = 0 (3.18)

At the side, the electric field is parallel to the area vector, thus

E · dA = EdA (3.19)

Thus the flux becomes,

Φ =

∫side sirface

E · dA = E∫

dA (3.20)

The area in this case is the surface area of the side of the cylinder, 2πrh.

Φ = 2πrhE (3.21)

Applying Gauss’ Law, we see that Φ = q/ε0. The electric field becomes

E =λ

2πε0r(3.22)

3.1.6 Electric Field in a Solid Non-Conducting Sphere

Within our non-conducting sphere or radius, R, we will assume that the total charge,Q is evenly distributed throughout the sphere’s volume. So the charge density of oursphere is

ρ =QV

=Q

43πR3

(3.23)

©2009 David S. Latchman

Page 28: 48578900 GRE Physics 8677 Solutions

DRAFT

28 ElectromagnetismThe Electric Field due to a charge Q is

E =Q

4πε0r2 (3.24)

As the charge is evenly distributed throughout the sphere’s volume we can say thatthe charge density is

dq = ρdV (3.25)

where dV = 4πr2dr. We can use this to determine the field inside the sphere bysumming the effect of infinitesimally thin spherical shells

E =

∫ E

0dE =

∫ r

0

dq4πεr2

ε0

∫ r

0dr

=Qr

43πε0R3

(3.26)

3.1.7 Electric Potential Energy

U(r) =1

4πε0qq0r (3.27)

3.1.8 Electric Potential of a Point Charge

The electrical potential is the potential energy per unit charge that is associated with astatic electrical field. It can be expressed thus

U(r) = qV(r) (3.28)

And we can see that

V(r) =1

4πε0

qr

(3.29)

A more proper definition that includes the electric field, E would be

V(r) = −

∫C

E · d` (3.30)

where C is any path, starting at a chosen point of zero potential to our desired point.The difference between two potentials can be expressed such

V(b) − V(a) = −

∫ b

E · d` +

∫ a

E · d`

= −

∫ b

aE · d` (3.31)

David S. Latchman ©2009

Page 29: 48578900 GRE Physics 8677 Solutions

DRAFT

Electrostatics 29This can be further expressed

V(b) − V(a) =

∫ b

a(∇V) · d` (3.32)

And we can show thatE = −∇V (3.33)

3.1.9 Electric Potential due to a line charge along axis

Let us consider a rod of length, `, with linear charge density, λ. The Electrical Potentialdue to a continuous distribution is

V =

∫dV =

14πε0

∫dqr

(3.34)

The charge density isdq = λdx (3.35)

Substituting this into the above equation, we get the electrical potential at some distancex along the rod’s axis, with the origin at the start of the rod.

dV =1

4πε0

dqx

=1

4πε0

λdxx

(3.36)

This becomes

V =λ

4πε0ln

[x2

x1

](3.37)

where x1 and x2 are the distances from O, the end of the rod.Now consider that we are some distance, y, from the axis of the rod of length, `. We

again look at Equation 3.34, where r is the distance of the point P from the rod’s axis.

V =1

4πε0

∫dqr

=1

4πε0

∫ `

0

λdx(x2 + y2

) 12

4πε0ln

[x +

(x2 + y2

) 12]`

0

4πε0ln

[` +

(`2 + y2

) 12]− ln y

4πε0ln

` +(`2 + y2) 1

2

d

(3.38)

©2009 David S. Latchman

Page 30: 48578900 GRE Physics 8677 Solutions

DRAFT

30 Electromagnetism3.2 Currents and DC Circuits

2

3.3 Magnetic Fields in Free Space

3

3.4 Lorentz Force

4

3.5 Induction

5

3.6 Maxwell’s Equations and their Applications

6

3.7 Electromagnetic Waves

7

3.8 AC Circuits

8

3.9 Magnetic and Electric Fields in Matter

9

3.10 Capacitance

Q = CV (3.39)

David S. Latchman ©2009

Page 31: 48578900 GRE Physics 8677 Solutions

DRAFT

Energy in a Capacitor 313.11 Energy in a Capacitor

U =Q2

2C

=CV2

2

=QV

2(3.40)

3.12 Energy in an Electric Field

u ≡U

volume=ε0E2

2(3.41)

3.13 Current

I ≡dQdt

(3.42)

3.14 Current Destiny

I =

∫A

J · dA (3.43)

3.15 Current Density of Moving Charges

J =IA

= neqvd (3.44)

3.16 Resistance and Ohm’s Law

R ≡VI

(3.45)

3.17 Resistivity and Conductivity

R = ρLA

(3.46)

E = ρJ (3.47)

J = σE (3.48)

©2009 David S. Latchman

Page 32: 48578900 GRE Physics 8677 Solutions

DRAFT

32 Electromagnetism3.18 Power

P = VI (3.49)

3.19 Kirchoff’s Loop Rules

Write Here

3.20 Kirchoff’s Junction Rule

Write Here

3.21 RC Circuits

E − IR −QC

= 0 (3.50)

3.22 Maxwell’s Equations

3.22.1 Integral Form

Gauss’ Law for Electric Fields

w

closed surface

E · dA =Qε0

(3.51)

Gauss’ Law for Magnetic Fields

w

closed surface

B · dA = 0 (3.52)

Ampere’s Lawz

B · ds = µ0I + µ0ε0ddt

w

surface

E · dA (3.53)

Faraday’s Lawz

E · ds = −ddt

w

surface

B · dA (3.54)

David S. Latchman ©2009

Page 33: 48578900 GRE Physics 8677 Solutions

DRAFT

Speed of Propagation of a Light Wave 333.22.2 Differential Form

Gauss’ Law for Electric Fields∇ · E =

ρ

ε0(3.55)

Gauss’ Law for Magnetism∇ · B = 0 (3.56)

Ampere’s Law

∇ × B = µ0J + µ0ε0∂E∂t

(3.57)

Faraday’s Law

∇ · E = −∂B∂t

(3.58)

3.23 Speed of Propagation of a Light Wave

c =1√µ0ε0

(3.59)

In a material with dielectric constant, κ,

c√κ =

cn

(3.60)

where n is the refractive index.

3.24 Relationship between E and B Fields

E = cB (3.61)E · B = 0 (3.62)

3.25 Energy Density of an EM wave

u =12

(B2

µ0+ ε0E2

)(3.63)

3.26 Poynting’s Vector

S =1µ0

E × B (3.64)

©2009 David S. Latchman

Page 34: 48578900 GRE Physics 8677 Solutions

DRAFT

34 Electromagnetism

David S. Latchman ©2009

Page 35: 48578900 GRE Physics 8677 Solutions

DRAFTChapter 4

Optics & Wave Phonomena

4.1 Wave Properties

1

4.2 Superposition

2

4.3 Interference

3

4.4 Diffraction

4

4.5 Geometrical Optics

5

4.6 Polarization

6

4.7 Doppler Effect

7

Page 36: 48578900 GRE Physics 8677 Solutions

DRAFT

36 Optics & Wave Phonomena4.8 Snell’s Law

4.8.1 Snell’s Law

n1 sinθ1 = n2 sinθ2 (4.1)

4.8.2 Critical Angle and Snell’s Law

The critical angle, θc, for the boundary seperating two optical media is the smallestangle of incidence, in the medium of greater index, for which light is totally refelected.

From Equation 4.1, θ1 = 90 and θ2 = θc and n2 > n1.

n1 sin 90 = n2sinθc

sinθc =n1

n2(4.2)

David S. Latchman ©2009

Page 37: 48578900 GRE Physics 8677 Solutions

DRAFTChapter 5

Thermodynamics & StatisticalMechanics

5.1 Laws of Thermodynamics

1

5.2 Thermodynamic Processes

2

5.3 Equations of State

3

5.4 Ideal Gases

4

5.5 Kinetic Theory

5

5.6 Ensembles

6

Page 38: 48578900 GRE Physics 8677 Solutions

DRAFT

38 Thermodynamics & Statistical Mechanics5.7 Statistical Concepts and Calculation of Thermody-

namic Properties

7

5.8 Thermal Expansion & Heat Transfer

8

5.9 Heat Capacity

Q = C(T f − Ti

)(5.1)

where C is the Heat Capacity and T f and Ti are the final and initial temperaturesrespectively.

5.10 Specific Heat Capacity

Q = cm(T f − ti

)(5.2)

where c is the specific heat capacity and m is the mass.

5.11 Heat and Work

W =

∫ V f

Vi

PdV (5.3)

5.12 First Law of Thermodynamics

dEint = dQ − dW (5.4)

where dEint is the internal energy of the system, dQ is the Energy added to the systemand dW is the work done by the system.

5.12.1 Special Cases to the First Law of Thermodynamics

Adiabatic Process During an adiabatic process, the system is insulated such that thereis no heat transfer between the system and its environment. Thus dQ = 0, so

∆Eint = −W (5.5)

If work is done on the system, negative W, then there is an increase in its internalenergy. Conversely, if work is done by the system, positive W, there is a decreasein the internal energy of the system.

David S. Latchman ©2009

Page 39: 48578900 GRE Physics 8677 Solutions

DRAFT

Work done by Ideal Gas at Constant Temperature 39Constant Volume (Isochoric) Process If the volume is held constant, then the system

can do no work, δW = 0, thus∆Eint = Q (5.6)

If heat is added to the system, the temperature increases. Conversely, if heat isremoved from the system the temperature decreases.

Closed Cycle In this situation, after certain interchanges of heat and work, the systemcomes back to its initial state. So ∆Eint remains the same, thus

∆Q = ∆W (5.7)

The work done by the system is equal to the heat or energy put into it.

Free Expansion In this process, no work is done on or by the system. Thus ∆Q =∆W = 0,

∆Eint = 0 (5.8)

5.13 Work done by Ideal Gas at Constant Temperature

Starting with Equation 5.3, we substitute the Ideal gas Law, Equation 5.11, to get

W = nRT∫ V f

Vi

dVV

= nRT lnV f

Vi(5.9)

5.14 Heat Conduction Equation

The rate of heat transferred, H, is given by

H =Qt

= kATH − TC

L(5.10)

where k is the thermal conductivity.

5.15 Ideal Gas Law

PV = nRT (5.11)

where

n = Number of molesP = PressureV = VolumeT = Temperature

©2009 David S. Latchman

Page 40: 48578900 GRE Physics 8677 Solutions

DRAFT

40 Thermodynamics & Statistical Mechanicsand R is the Universal Gas Constant, such that

R ≈ 8.314 J/mol. K

We can rewrite the Ideal gas Law to say

PV = NkT (5.12)

where k is the Boltzmann’s Constant, such that

k =R

NA≈ 1.381 × 10−23 J/K

5.16 Stefan-Boltzmann’s Formula

P(T) = σT4 (5.13)

5.17 RMS Speed of an Ideal Gas

vrms =

√3RTM

(5.14)

5.18 Translational Kinetic Energy

K =32

kT (5.15)

5.19 Internal Energy of a Monatomic gas

Eint =32

nRT (5.16)

5.20 Molar Specific Heat at Constant Volume

Let us define, CV such thatQ = nCV∆T (5.17)

Substituting into the First Law of Thermodynamics, we have

∆Eint + W = nCV∆T (5.18)

At constant volume, W = 0, and we get

CV =1n

∆Eint

∆T(5.19)

Substituting (Equation 5.16), we get

CV =32

R = 12.5 J/mol.K (5.20)

David S. Latchman ©2009

Page 41: 48578900 GRE Physics 8677 Solutions

DRAFT

Molar Specific Heat at Constant Pressure 415.21 Molar Specific Heat at Constant Pressure

Starting withQ = nCp∆T (5.21)

and

∆Eint = Q −W⇒ nCV∆T = nCp∆T + nR∆T

∴ CV = Cp − R (5.22)

5.22 Equipartition of Energy

CV =

(f2

)R = 4.16 f J/mol.K (5.23)

where f is the number of degrees of freedom.

Table 5.1: Table of Molar Specific Heats

Degrees of Freedom Predicted Molar Specific HeatsMolecule Translational Rotational Total ( f ) CV CP = CV + RMonatomic 3 0 3 3

2R 52R

Diatomic 3 2 5 52R 7

2RPolyatomic 3 3 6 3R 4R

5.23 Adiabatic Expansion of an Ideal Gas

PVγ = a constant (5.24)

where γ = CPCV

.We can also write

TVγ−1 = a constant (5.25)

5.24 Second Law of Thermodynamics

Something.

©2009 David S. Latchman

Page 42: 48578900 GRE Physics 8677 Solutions

DRAFT

42 Thermodynamics & Statistical Mechanics

David S. Latchman ©2009

Page 43: 48578900 GRE Physics 8677 Solutions

DRAFTChapter 6

Quantum Mechanics

6.1 Fundamental Concepts

1

6.2 Schrodinger Equation

Let us define Ψ to beΨ = Ae−iω(t− x

v ) (6.1)

Simplifying in terms of Energy, E, and momentum, p, we get

Ψ = Ae−i(Et−px)~ (6.2)

We obtain Schrodinger’s Equation from the Hamiltonian

H = T + V (6.3)

To determine E and p,

∂2Ψ

∂x2 = −p2

~2 Ψ (6.4)

∂Ψ∂t

=iE~

Ψ (6.5)

and

H =p2

2m+ V (6.6)

This becomes

EΨ = HΨ (6.7)

EΨ = −~

i∂Ψ∂t

p2Ψ = −~2∂2Ψ

∂x2

Page 44: 48578900 GRE Physics 8677 Solutions

DRAFT

44 Quantum MechanicsThe Time Dependent Schrodinger’s Equation is

i~∂Ψ∂t

= −~2

2m∂2Ψ

∂x2 + V(x)Ψ (6.8)

The Time Independent Schrodinger’s Equation is

EΨ = −~2

2m∂2Ψ

∂x2 + V(x)Ψ (6.9)

6.2.1 Infinite Square Wells

Let us consider a particle trapped in an infinite potential well of size a, such that

V(x) =

0 for 0 < x < a∞ for |x| > a,

so that a nonvanishing force acts only at ±a/2. An energy, E, is assigned to the systemsuch that the kinetic energy of the particle is E. Classically, any motion is forbiddenoutside of the well because the infinite value of V exceeds any possible choice of E.

Recalling the Schrodinger Time Independent Equation, Equation 6.9, we substituteV(x) and in the region (−a/2, a/2), we get

−~2

2md2ψ

dx2 = Eψ (6.10)

This differential is of the formd2ψ

dx2 + k2ψ = 0 (6.11)

where

k =

√2mE~2 (6.12)

We recognize that possible solutions will be of the form

cos kx and sin kx

As the particle is confined in the region 0 < x < a, we say

ψ(x) =

A cos kx + B sin kx for 0 < x < a0 for |x| > a

We have known boundary conditions for our square well.

ψ(0) = ψ(a) = 0 (6.13)

It shows that

⇒ A cos 0 + B sin 0 = 0∴ A = 0 (6.14)

David S. Latchman ©2009

Page 45: 48578900 GRE Physics 8677 Solutions

DRAFT

Schrodinger Equation 45We are now left with

B sin ka = 0ka = 0;π; 2π; 3π; · · ·

(6.15)

While mathematically, n can be zero, that would mean there would be no wave function,so we ignore this result and say

kn =nπa

for n = 1, 2, 3, · · ·

Substituting this result into Equation 6.12 gives

kn =nπa

=

√2mEn

~(6.16)

Solving for En gives

En =n2π2~2

2ma2 (6.17)

We cna now solve for B by normalizing the function∫ a

0|B|2 sin2 kxdx = |A|2

a2

= 1

So |A|2 =2a

(6.18)

So we can write the wave function as

ψn(x) =

√2a

sin(nπx

a

)(6.19)

6.2.2 Harmonic Oscillators

Classically, the harmonic oscillator has a potential energy of

V(x) =12

kx2 (6.20)

So the force experienced by this particle is

F = −dVdx

= −kx (6.21)

where k is the spring constant. The equation of motion can be summed us as

md2xdt2 = −kx (6.22)

And the solution of this equation is

x(t) = A cos(ω0t + φ

)(6.23)

©2009 David S. Latchman

Page 46: 48578900 GRE Physics 8677 Solutions

DRAFT

46 Quantum Mechanicswhere the angular frequency, ω0 is

ω0 =

√km

(6.24)

The Quantum Mechanical description on the harmonic oscillator is based on the eigen-function solutions of the time-independent Schrodinger’s equation. By taking V(x)from Equation 6.20 we substitute into Equation 6.9 to get

d2ψ

dx2 =2m~2

(k2

x2− E

)ψ =

mk~2

(x2−

2Ek

With some manipulation, we get

~√

mk

d2ψ

dx2 =

√mk~

x2−

2E~

√mk

ψThis step allows us to to keep some of constants out of the way, thus giving us

ξ2 =

√mk~

x2 (6.25)

and λ =2E~

√mk

=2E~ω0

(6.26)

This leads to the more compact

d2ψ

dξ2 =(ξ2− λ

)ψ (6.27)

where the eigenfunction ψ will be a function of ξ. λ assumes an eigenvalue anaglaousto E.

From Equation 6.25, we see that the maximum value can be determined to be

ξ2max =

√mk~

A2 (6.28)

Using the classical connection between A and E, allows us to say

ξ2max =

√mk~

2Ek

= λ (6.29)

From Equation 6.27, we see that in a quantum mechanical oscillator, there are non-vanishing solutions in the forbidden regions, unlike in our classical case.

A solution to Equation 6.27 is

ψ(ξ) = e−ξ2/2 (6.30)

wheredψdξ

= −ξe−ξ2/2

anddψ

dξ2 = ξ2e−xi2/2− e−ξ

2/2 =(ξ2− 1

)e−ξ

2/2

David S. Latchman ©2009

Page 47: 48578900 GRE Physics 8677 Solutions

DRAFT

Schrodinger Equation 47This gives is a special solution for λ where

λ0 = 1 (6.31)

Thus Equation 6.26 gives the energy eigenvalue to be

E0 =~ω0

2λ0 =

~ω0

2(6.32)

The eigenfunction e−ξ2/2 corresponds to a normalized stationary-state wave function

Ψ0(x, t) =

(mkπ2~2

) 18

e−√

mkx2/2~e−iE0t/~ (6.33)

This solution of Equation 6.27 produces the smallest possibel result of λ and E. Hence,Ψ0 and E0 represents the ground state of the oscillator. and the quantity ~ω0/2 is thezero-point energy of the system.

6.2.3 Finite Square Well

For the Finite Square Well, we have a potential region where

V(x) =

−V0 for −a ≤ x ≤ a0 for |x| > a

We have three regions

Region I: x < −a In this region, The potential, V = 0, so Schrodinger’s Equation be-comes

−~2

2md2ψ

dx2 = Eψ

⇒d2ψ

dx2 = κ2ψ

where κ =

√−2mE~

This gives us solutions that are

ψ(x) = A exp(−κx) + B exp(κx)

As x → ∞, the exp(−κx) term goes to ∞; it blows up and is not a physicallyrealizable function. So we can drop it to get

ψ(x) = Beκx for x < −a (6.34)

©2009 David S. Latchman

Page 48: 48578900 GRE Physics 8677 Solutions

DRAFT

48 Quantum MechanicsRegion II: −a < x < a In this region, our potential is V(x) = V0. Substitutin this into

the Schrodinger’s Equation,Equation 6.9, gives

−~2

2md2ψ

dx2 − V0ψ = Eψ

ord2ψ

dx2 = −l2ψ

where l ≡

√2m (E + V0)~

(6.35)

We notice that E > −V0, making l real and positive. Thus our general solutionbecomes

ψ(x) = C sin(lx) + D cos(lx) for −a < x < a (6.36)

Region III: x > a Again this Region is similar to Region III, where the potential, V = 0.This leaves us with the general solution

ψ(x) = F exp(−κx) + G exp(κx)

As x→∞, the second term goes to infinity and we get

ψ(x) = Fe−κx for x > a (6.37)

This gives us

ψ(x) =

Beκx for x < aD cos(lx) for 0 < x < aFe−κx for x > a

(6.38)

6.2.4 Hydrogenic Atoms

c

6.3 Spin

3

6.4 Angular Momentum

4

6.5 Wave Funtion Symmetry

5

6.6 Elementary Perturbation Theory

6

David S. Latchman ©2009

Page 49: 48578900 GRE Physics 8677 Solutions

DRAFTChapter 7

Atomic Physics

7.1 Properties of Electrons

1

7.2 Bohr Model

To understand the Bohr Model of the Hydrogen atom, we will take advantage of ourknowlegde of the wavelike properties of matter. As we are building on a classicalmodel of the atom with a modern concept of matter, our derivation is considered to be‘semi-classical’. In this model we have an electron of mass, me, and charge, −e, orbitinga proton. The cetripetal force is equal to the Coulomb Force. Thus

14πε0

e2

r2 =mev2

r(7.1)

The Total Energy is the sum of the potential and kinetic energies, so

E = K + U =p2

2me− | f race24πε0r (7.2)

We can further reduce this equation by subsituting the value of momentum, which wefind to be

p2

2me=

12

mev2 =e2

8πε0r(7.3)

Substituting this into Equation 7.2, we get

E =e2

8πε0r−

e2

4πε0r= −

e2

8πε0r(7.4)

At this point our classical description must end. An accelerated charged particle, likeone moving in circular motion, radiates energy. So our atome here will radiate energyand our electron will spiral into the nucleus and disappear. To solve this conundrum,Bohr made two assumptions.

Page 50: 48578900 GRE Physics 8677 Solutions

DRAFT

50 Atomic Physics1. The classical circular orbits are replaced by stationary states. These stationary

states take discreet values.

2. The energy of these stationary states are determined by their angular momentumwhich must take on quantized values of ~.

L = n~ (7.5)

We can find the angular momentum of a circular orbit.

L = m3vr (7.6)

From Equation 7.1 we find v and by substitution, we find L.

L = e√

m3r4πε0

(7.7)

Solving for r, gives

r =L2

mee2/4πε0(7.8)

We apply the condition from Equation 7.5

rn =n2~2

mee2/4πε0= n2a0 (7.9)

where a0 is the Bohr radius.a0 = 0.53 × 10−10 m (7.10)

Having discreet values for the allowed radii means that we will also have discreetvalues for energy. Replacing our value of rn into Equation 7.4, we get

En = −me

2n2

(e2

4πε0~

)= −

13.6n2 eV (7.11)

7.3 Energy Quantization

3

7.4 Atomic Structure

4

David S. Latchman ©2009

Page 51: 48578900 GRE Physics 8677 Solutions

DRAFT

Atomic Spectra 517.5 Atomic Spectra

7.5.1 Rydberg’s Equation1λ

= RH

( 1n′2−

1n2

)(7.12)

where RH is the Rydberg constant.For the Balmer Series, n′ = 2, which determines the optical wavelengths. For

n′ = 3, we get the infrared or Paschen series. The fundamental n′ = 1 series falls in theultraviolet region and is known as the Lyman series.

7.6 Selection Rules

6

7.7 Black Body Radiation

7.7.1 Plank Formula

u( f ,T) =8π~c3

f 3

eh f/kT − 1(7.13)

7.7.2 Stefan-Boltzmann Formula

P(T) = σT4 (7.14)

7.7.3 Wein’s Displacement Law

λmaxT = 2.9 × 10−3 m.K (7.15)

7.7.4 Classical and Quantum Aspects of the Plank Equation

Rayleigh’s Equation

u( f ,T) =8π f 2

c3 kT (7.16)

We can get this equation from Plank’s Equation, Equation 7.13. This equation is aclassical one and does not contain Plank’s constant in it. For this case we will look atthe situation where h f < kT. In this case, we make the approximation

ex' 1 + x (7.17)

Thus the demonimator in Equation 7.13 becomes

eh f/kT− 1 ' 1 +

h fkT− 1 =

h fkT

(7.18)

©2009 David S. Latchman

Page 52: 48578900 GRE Physics 8677 Solutions

DRAFT

52 Atomic PhysicsThus Equation 7.13 takes the approximate form

u( f ,T) '8πhc3 f 3 kT

h f=

8π f 2

c3 kT (7.19)

As we can see this equation is devoid of Plank’s constant and thus independent ofquantum effects.

Quantum

At large frequencies, where h f > kT, quantum effects become apparent. We canestimate that

eh f/kT− 1 ' eh f/kT (7.20)

Thus Equation 7.13 becomes

u( f ,T) '8πhc3 f 3e−h f/kT (7.21)

7.8 X-Rays

7.8.1 Bragg Condition

2d sinθ = mλ (7.22)

for constructive interference off parallel planes of a crystal with lattics spacing, d.

7.8.2 The Compton Effect

The Compton Effect deals with the scattering of monochromatic X-Rays by atomictargets and the observation that the wavelength of the scattered X-ray is greater thanthe incident radiation. The photon energy is given by

E = hυ =hcλ

(7.23)

The photon has an associated momentum

E = pc (7.24)

⇒ p =E

c=

hυc

=hλ

(7.25)

The Relativistic Energy for the electron is

E2 = p2c2 + m2e c4 (7.26)

wherep − p′ = P (7.27)

David S. Latchman ©2009

Page 53: 48578900 GRE Physics 8677 Solutions

DRAFT

Atoms in Electric and Magnetic Fields 53Squaring (Equation 7.27) gives

p2− 2p · p′ + p′2 = P2 (7.28)

Recall that E = pc and E ′ = cp′, we have

c2p2− 2c2p · p′ + c2p′2 = c2P2

E 2− 2E E ′ cosθ + E ′2 = E2

−m2e c4 (7.29)

Conservation of Energy leads to

E + mec2 = E ′ + E (7.30)

Solving

E − E ′ = E −mec2

E 2− 2E E ′ + E ′ = E2

− 2Emec2 + m2e c4 (7.31)

2E E ′ − 2E E ′ cosθ = 2Emec2− 2m2

e c4 (7.32)

Solving leads to

∆λ = λ′ − λ =h

mec(1 − cosθ) (7.33)

where λc = hmec

is the Compton Wavelength.

λc =h

mec= 2.427 × 10−12m (7.34)

7.9 Atoms in Electric and Magnetic Fields

7.9.1 The Cyclotron Frequency

A test charge, q, with velocity v enters a uniform magnetic field, B. The force acting onthe charge will be perpendicular to v such that

FB = qv × B (7.35)

or more simply FB = qvB. As this traces a circular path, from Newton’s Second Law,we see that

FB =mv2

R= qvB (7.36)

Solving for R, we getR =

mvqB

(7.37)

We also see that

f =qB

2πm(7.38)

The frequency is depends on the charge, q, the magnetic field strength, B and the massof the charged particle, m.

©2009 David S. Latchman

Page 54: 48578900 GRE Physics 8677 Solutions

DRAFT

54 Atomic Physics7.9.2 Zeeman Effect

The Zeeman effect was the splitting of spectral lines in a static magnetic field. This issimilar to the Stark Effect which was the splitting in the presence in a magnetic field.

In the Zeeman experiment, a sodium flame was placed in a magnetic field and itsspectrum observed. In the presence of the field, a spectral line of frequency, υ0 wassplit into three components, υ0 − δυ, υ0 and υ0 + δυ. A classical analysis of this effectallows for the identification of the basic parameters of the interacting system.

The application of a constant magnetic field, B, allows for a direction in space inwhich the electron motion can be referred. The motion of an electron can be attributedto a simple harmonic motion under a binding force −kr, where the frequency is

υ0 =1

√k

me(7.39)

The magnetic field subjects the electron to an additional Lorentz Force, −ev × B. Thisproduces two different values for the angular velocity.

v = 2πrυ

The cetripetal force becomesmev2

r= 4π2υ2rme

Thus the certipetal force is

4π2υ2rme = 2πυreB + kr for clockwise motion

4π2υ2rme = −2πυreB + kr for counterclockwise motion

We use Equation 7.39, to emiminate k, to get

υ2−

eB2πme

υ − υ0 = 0 (Clockwise)

υ2 +eB

2πmeυ − υ0 = 0 (Counterclockwise)

As we have assumed a small Lorentz force, we can say that the linear terms in υ aresmall comapred to υ0. Solving the above quadratic equations leads to

υ = υ0 +eB

4πmefor clockwise motion (7.40)

υ = υ0 −eB

4πmefor counterclockwise motion (7.41)

We note that the frequency shift is of the form

δυ =eB

4πme(7.42)

If we view the source along the direction of B, we will observe the light to have twopolarizations, a closckwise circular polarization of υ0 + δυ and a counterclosckwisecircular polarization of υ0 − δυ.

David S. Latchman ©2009

Page 55: 48578900 GRE Physics 8677 Solutions

DRAFT

Atoms in Electric and Magnetic Fields 557.9.3 Franck-Hertz Experiment

The Franck-Hertz experiment, performed in 1914 by J. Franck and G. L. Hertz, mea-sured the colisional excitation of atoms. Their experiement studied the current ofelectrons in a tub of mercury vapour which revealed an abrupt change in the currentat certain critical values of the applied voltage.1 They interpreted this observation asevidence of a threshold for inelastic scattering in the colissions of electrons in mer-cury atoms.The bahavior of the current was an indication that electrons could losea discreet amount of energy and excite mercury atoms in their passage through themercury vapour. These observations constituted a direct and decisive confirmation ofthe existence os quantized energy levels in atoms.

1Put drawing of Franck-Hertz Setup

©2009 David S. Latchman

Page 56: 48578900 GRE Physics 8677 Solutions

DRAFT

56 Atomic Physics

David S. Latchman ©2009

Page 57: 48578900 GRE Physics 8677 Solutions

DRAFTChapter 8

Special Relativity

8.1 Introductory Concepts

8.1.1 Postulates of Special Relativity

1. The laws of Physics are the same in all inertial frames.

2. The speed of light is the same in all inertial frames.

We can define

γ =1√

1 − u2

c2

(8.1)

8.2 Time Dilation

∆t = γ∆t′ (8.2)

where ∆t′ is the time measured at rest relative to the observer, ∆t is the time measuredin motion relative to the observer.

8.3 Length Contraction

L =L′

γ(8.3)

where L′ is the length of an object observed at rest relative to the observer and L is thelength of the object moving at a speed u relative to the observer.

8.4 Simultaneity

4

Page 58: 48578900 GRE Physics 8677 Solutions

DRAFT

58 Special Relativity8.5 Energy and Momentum

8.5.1 Relativistic Momentum & Energy

In relativistic mechanics, to be conserved, momentum and energy are defined as

Relativistic Momentum

p = γmv (8.4)

Relativistic Energy

E = γmc2 (8.5)

8.5.2 Lorentz Transformations (Momentum & Energy)

p′x = γ(px − β

Ec

)(8.6)

p′y = py (8.7)

p′z = pz (8.8)E′

c= γ

(Ec− βpx

)(8.9)

8.5.3 Relativistic Kinetic Energy

K = E −mc2 (8.10)

= mc2

1√1 − v2

c2

− 1

(8.11)

= mc2 (γ − 1)

(8.12)

8.5.4 Relativistic Dynamics (Collisions)

∆P′x = γ(∆Px − β

∆Ec

)(8.13)

∆P′y = ∆Py (8.14)

∆P′z = ∆Pz (8.15)∆E′

c= γ

(∆Ec− β∆Px

)(8.16)

David S. Latchman ©2009

Page 59: 48578900 GRE Physics 8677 Solutions

DRAFT

Four-Vectors and Lorentz Transformation 598.6 Four-Vectors and Lorentz Transformation

We can represent an event in S with the column matrix, s,

s =

xyz

ict

(8.17)

A different Lorents frame, S′, corresponds to another set of space time axes so that

s′ =

x′

y′

z′

ict′

(8.18)

The Lorentz Transformation is related by the matrixx′

y′

z′

ict′

=

γ 0 0 iγβ0 1 0 00 0 1 0−iγβ 0 0 γ

xyz

ict

(8.19)

We can express the equation in the form

s′ = L s (8.20)

The matrix L contains all the information needed to relate position four–vectors forany given event as observed in the two Lorentz frames S and S′. If we evaluate

sTs =[

x y z ict]

xyz

ict

= x2 + y2 + z2− c2t2 (8.21)

Similarly we can show that

s′Ts′ = x′2 + y′2 + z′2 − c2t′2 (8.22)

We can take any collection of four physical quantities to be four vector provided thatthey transform to another Lorentz frame. Thus we have

b =

bx

by

bz

ibt

(8.23)

this can be transformed into a set of quantities of b′ in another frame S′ such that itsatisfies the transformation

b′ = L b (8.24)

©2009 David S. Latchman

Page 60: 48578900 GRE Physics 8677 Solutions

DRAFT

60 Special RelativityLooking at the momentum-Energy four vector, we have

p =

px

py

pz

iE/c

(8.25)

Applying the same transformation rule, we have

p′ = L p (8.26)

We can also get a Lorentz-invariation relation between momentum and energy suchthat

p′Tp′ = pTp (8.27)

The resulting equality gives

p′2x + p′2y + p′2z −E′2

c2 = p2x + p2

y + p2z −

E2

c2 (8.28)

8.7 Velocity Addition

v′ =v − u1 − uv

c2

(8.29)

8.8 Relativistic Doppler Formula

υ = υ0

√c + uc − u

let r =

√c − uc + u

(8.30)

We have

υreceding = rυ0 red-shift (Source Receding) (8.31)

υapproaching =υ0

rblue-shift (Source Approaching) (8.32)

8.9 Lorentz Transformations

Given two reference frames S(x, y, z, t) and S′(x′, y′, z′, t′), where the S′-frame is movingin the x-direction, we have,

x′ = γ (x − ut) x = (x′ − ut′) (8.33)y′ = y y = y′ (8.34)z′ = y y′ = y (8.35)

t′ = γ(t −

uc2 x

)t = γ

(t′ +

uc2 x′

)(8.36)

David S. Latchman ©2009

Page 61: 48578900 GRE Physics 8677 Solutions

DRAFT

Space-Time Interval 618.10 Space-Time Interval

(∆S)2 = (∆x)2 +(∆y

)2+ (∆z)2

− c2 (∆t)2 (8.37)

Space-Time Intervals may be categorized into three types depending on their separa-tion. They are

Time-like Interval

c2∆t2 > ∆r2 (8.38)

∆S2 > 0 (8.39)

When two events are separated by a time-like interval, there is a cause-effectrelationship between the two events.

Light-like Interval

c2∆t2 = ∆r2 (8.40)

S2 = 0 (8.41)

Space-like Intervals

c2∆t2 < ∆r2 (8.42)∆S < 0 (8.43)

©2009 David S. Latchman

Page 62: 48578900 GRE Physics 8677 Solutions

DRAFT

62 Special Relativity

David S. Latchman ©2009

Page 63: 48578900 GRE Physics 8677 Solutions

DRAFTChapter 9

Laboratory Methods

9.1 Data and Error Analysis

9.1.1 Addition and Subtraction

x = a + b − c (9.1)

The Error in x is(δx)2 = (δa)2 + (δb)2 + (δc)2 (9.2)

9.1.2 Multiplication and Division

x =a × b

c(9.3)

The error in x is (δxx

)2

=(δaa

)2

+

(δbb

)2

+(δcc

)2

(9.4)

9.1.3 Exponent - (No Error in b)

x = ab (9.5)

The Error in x isδxx

= b(δaa

)(9.6)

9.1.4 Logarithms

Base e

x = ln a (9.7)

Page 64: 48578900 GRE Physics 8677 Solutions

DRAFT

64 Laboratory MethodsWe find the error in x by taking the derivative on both sides, so

δx =d ln a

da· δa

=1a· δa

=δaa

(9.8)

Base 10

x = log10 a (9.9)The Error in x can be derived as such

δx =d(log a)

daδa

=ln a

ln 10

daδa

=1

ln 10δaa

= 0.434δaa

(9.10)

9.1.5 Antilogs

Base e

x = ea (9.11)We take the natural log on both sides.

ln x = a ln e = a (9.12)

Applaying the same general method, we seed ln x

dxδx = δa

⇒δxx

= δa (9.13)

Base 10

x = 10a (9.14)We follow the same general procedure as above to get

log x = a log 10log x

dxδx = δa

1ln 10

d ln adx

δx = δa

δxx

= ln 10δa (9.15)

David S. Latchman ©2009

Page 65: 48578900 GRE Physics 8677 Solutions

DRAFT

Instrumentation 659.2 Instrumentation

2

9.3 Radiation Detection

3

9.4 Counting Statistics

Let’s assume that for a particular experiment, we are making countung measurementsfor a radioactive source. In this experiment, we recored N counts in time T. Thecounting rate for this trial is R = N/T. This rate should be close to the average rate, R.The standard deviation or the uncertainty of our count is a simply called the

√N rule.

Soσ =√

N (9.16)

Thus we can report our results as

Number of counts = N ±√

N (9.17)

We can find the count rate by dividing by T, so

R =NT±

√N

T(9.18)

The fractional uncertainty of our count is δNN . We can relate this in terms of the count

rate.

δRR

=δNTNT

=δNN

=

√N

N

=1N

(9.19)

We see that our uncertainty decreases as we take more counts, as to be expected.

9.5 Interaction of Charged Particles with Matter

5

9.6 Lasers and Optical Interferometers

6

©2009 David S. Latchman

Page 66: 48578900 GRE Physics 8677 Solutions

DRAFT

66 Laboratory Methods9.7 Dimensional Analysis

Dimensional Analysis is used to understand physical situations involving a mis ofdifferent types of physical quantities. The dimensions of a physical quantity areassociated with combinations of mass, length, time, electric charge, and temperature,represented by symbols M, L, T, Q, and θ, respectively, each raised to rational powers.

9.8 Fundamental Applications of Probability and Statis-tics

8

David S. Latchman ©2009

Page 67: 48578900 GRE Physics 8677 Solutions

DRAFTChapter 10

GR8677 Exam Solutions

10.1 Motion of Rock under Drag Force

From the information provided we can come up with an equation of motion for therock.

mx = −mg − kv (10.1)

If you have seen this type of equation, and solved it, you’d know that the rock’s speedwill asymtotically increase to some max speed. At that point the drag force and theforce due to gravity will be the same. We can best answer this question through analysisand elimination.

A Dividing Equation 10.1 by m gives

x = −g −km

x (10.2)

We see that this only occurs when x = 0, which only happens at the top of theflight. So FALSE.

B From Equation 10.2, we see that this is TRUE.

C Again from Equation 10.2 we see that the acceleration is dependent on whether therock is moving up or down. If x > 0 then x < −g and if x < 0 then x > −g. So thisis also FALSE.

D If there was no drag (fictional) force, then energy would be conserved and the rockwill return at the speed it started with but there is a drag force so energy is lost.The speed the rock returns is v < v0. Hence FALSE.

E Again FALSE. Once the drag force and the gravitational force acting on the rock isbalanced the rock won’t accelerate.

Answer: (B)

Page 68: 48578900 GRE Physics 8677 Solutions

DRAFT

68 GR8677 Exam Solutions10.2 Satellite Orbits

The question states that the astronaut fires the rocket’s jets towards Earth’s center. Weinfer that no extra energy is given to the system by this process. subsection 2.7.5, showsthat the only other orbit where the specific energy is also negative is an elliptical one.

Answer: (A)

10.3 Speed of Light in a Dielectric Medium

Solutions to the Electromagnetic wave equation gives us the speed of light in terms ofthe electromagnetic permeability, µ0 and permitivitty, ε0.

c =1√µ0ε0

(10.3)

where c is the speed of light. The speed through a dielectric medium becomes

v =1√µε0

=1√

2.1µ0ε0

=c√

2.1(10.4)

Answer: (D)

10.4 Wave Equation

We are given the equation

y = A sin( tT−

)(10.5)

We can analyze and eliminate from what we know about this equation

A The Amplitude, A in the equation is the displacement from equilibrium. So thischoice is incorrect.

B As the wave moves, we seek to keep the(

tT −

)term constant. So as t increases, we

expect x to increase as well as there is a negative sign in front of it. This meansthat the wave moves in the positive x-direction. This choice is also incorrect.

C The phase of the wave is given by(

tT −

), we can do some manipulation to show( t

T−

)= 2π f t − kx

= ωt − kx (10.6)

David S. Latchman ©2009

Page 69: 48578900 GRE Physics 8677 Solutions

DRAFT

Inelastic Collision and Putty Spheres 69Or rather

kx = ωt (10.7)

Differentiating Equation 10.7 gives us the phase speed, which is

v =λT

(10.8)

This is also incorrect

E From Equation 10.8 the above we see that is the answer.

Answer: (E)

10.5 Inelastic Collision and Putty Spheres

We are told the two masses coalesce so we know that the collision is inelastic andhence, energy is not conserved. As mass A falls, it looses Potential Energy and gainsKinetic Energy.

Mgh0 =12

Mv20 (10.9)

Thusv2

0 = 2gh0 (10.10)

Upon collision, momentum is conserved, thus

Mv0 = (3M + M) v1

= 4Mv1

⇒ v1 =v0

4(10.11)

The fused putty mass rises, kinetic energy is converted to potential energy and we findour final height.

12

(4M) v21 = 4Mgh

h =v2

1

2g

=1

2g

(v0

4

)2

=h0

16(10.12)

Answer: (A)

©2009 David S. Latchman

Page 70: 48578900 GRE Physics 8677 Solutions

DRAFT

70 GR8677 Exam Solutions10.6 Motion of a Particle along a Track

As the particle moves from the top of the track and runs down the frictionless track,its Gravitational Potential Energy is converted to Kinetic Energy. Let’s assume that theparticle is at a height, y0 when x = 0. Since energy is conserved, we get1

mgy0 = mg(y0 − y) +12

mv2

⇒12

v2 = gy (10.13)

So we have a relationship between v and the particle’s position on the track.The tangential acceleration in this case is

mg cosθ =mv2

r(10.14)

where r is the radius of curvature and is equal to√

x2 + y2.Substituting this into Equation 10.14 gives

g cosθ =v2

r

=gx2

2√

x2 + y2

=gx

x2 + 4(10.15)

Answer: (D)

10.7 Resolving Force Components

This question is a simple matter of resolving the horizontal and vertical componentsof the tension along the rope. We have

T sinθ = F (10.16)T cosθ = mg (10.17)

Thus we get

tanθ =F

mg

=10

(2)(9.8)≈

12

(10.18)

Answer: (A)1Insert Free Body Diagram of particle along track.

David S. Latchman ©2009

Page 71: 48578900 GRE Physics 8677 Solutions

DRAFT

Nail being driven into a block of wood 7110.8 Nail being driven into a block of wood

We recall thatv2 = v2

0 + 2as (10.19)

where v, v0, a and s are the final speed, initial speed, acceleration and displacementthat the nail travels. Now it’s just a matter os plugging in what we know

0 = 100 + 2a(0.025) (10.20)

⇒ a = −100

2(0.025)= −2000 m/s2 (10.21)

The Force on the nail comes from Newton’s Second Law

F = ma= 5 · 2000 = 10000 N (10.22)

Answer: (D)

10.9 Current Density

We can find the drift vleocity from the current density equation

J = envd (10.23)

where e is the charge of an electron, n is the density of electrons per unit volume and~vd is the drift speed. Plugging in what we know

J =IA

I =nAvde

vd =I

nAe

=100

(1 × 1028)π×2×10−4

4 1.6 × 10−19(10.24)

paying attention to the indices of the equation we get

2 − 28 + 4 + 19 = −4 (10.25)

So we expect an answer where vd ≈ 10−4.2

Answer: (D)

2It also helps if you knew that the electron drift velocity was slow, in the order of mm/s.

©2009 David S. Latchman

Page 72: 48578900 GRE Physics 8677 Solutions

DRAFT

72 GR8677 Exam Solutions10.10 Charge inside an Isolated Sphere

You can answer this by thinking about Gauss’ Law. The bigger the Gaussian surfacethe more charge it encloses and the bigger the electric field. Beyond the radius of thesphere, the field decreases exponentially.

We can calculate these relationships by using Gauss’ Law.∮S

E · dS =Qenclosed

ε0(10.26)

where the current density, ρ is

ρ =Q

43πR3

=Qenclosed

43πr3

(10.27)

where R is the radius of the sphere.

for r < R The enclosed charge becomes

Qenclosed = ρ(43πr3

)=

Qr3

R3 (10.28)

Gauss’ Law becomes

E(4πr2

)=

Qr3

ε0R3 (10.29)

The Electric field is

E(r<R) =Qr

4πε0R3 (10.30)

This is a linear relationship with respect to r.

for r ≥ R The enclosed charge isQenclosed = Q (10.31)

Gauss’ Law becomes

E(4πr2

)=

Qε0

(10.32)

The Electric field is

E(r≥R) =Q

4πε0r2 (10.33)

The linear increase is exhibited by choice (C).Answer: (C)

David S. Latchman ©2009

Page 73: 48578900 GRE Physics 8677 Solutions

DRAFT

Vector Identities and Maxwell’s Laws 7310.11 Vector Identities and Maxwell’s Laws

We recall the vector identity∇ · (∇ ×A) = 0 (10.34)

Thus

∇ · (∇ ×H) = ∇ ·

(D + J

)= 0 (10.35)

Answer: (A)

10.12 Doppler Equation (Non-Relativistic)

we recall the Doppler Equation3

f = f0

(v − vr

v − vs

)(10.36)

where vr and vs are the observer and source speeds respectively. We are told that vr = 0and vs = 0.9v. Thus

f = f0

( vv − 0.9v

)= 10 f0

= 10 kHz (10.37)

Answer: (E)

10.13 Vibrating Interference Pattern

Answering this question takes some analysis. The sources are coherent, so they willproduce an interference pattern. We are also told that ∆ f = 500 Hz. This will producea shifting interference pattern that changes too fast for the eye to see.4

Answer: (E)

10.14 Specific Heat at Constant Pressure and Volume

From section 5.20 and section 5.21, we see that

Cp = CV + R (10.38)

The difference is due to the work done in the environment by the gas when it expandsunder constant pressure.

3Add reference to Dopler Equations.4Add Young’s Double Slit Experiment equations.

©2009 David S. Latchman

Page 74: 48578900 GRE Physics 8677 Solutions

DRAFT

74 GR8677 Exam SolutionsWe can prove this by starting with the First Law of Thermodynamics.

dU = −dW + dQ (10.39)

Where dU is the change in Internal Energy, dW is the work done by the system and dQis the change in heat of the system.

We also recall the definition for Heat Capacity

dQ = CdT (10.40)

At constant volume, there is no work done by the system, dV = 0. So it follows thatdW = 0. The change in internal energy is the change of heat into the system, thus wecan define, the heat capacity at constant volume to be

dUV = CVdT = dQV (10.41)

At constant pressure, the change in internal energy is accompanied by a change in heatflow as well as a change in the volume of the gas, thus

dUp = −dWp + dQp

= −pdV + CpdT where pdV = nRdT= −nRdT + CpdT (10.42)

If the changes in internal energies are the same in both cases, then Equation 10.42 isequal to Equation 10.41. Thus

CVdT = −nRdT + CpdT

This becomesCp = CV + nR (10.43)

We see the reason why Cp > CV is due to the addition of work on the system; Equa-tion 10.41 shows no work done by the gas while Equation 10.42 shows that there iswork done.

Answer: (A)

10.15 Helium atoms in a box

Let’s say the probability of the atoms being inside the box is 1. So the probability thatan atom will be found outside of a 1.0 × 10−6 cm3 box is

P = 1 − 1.0 × 10−6 (10.44)

As there are N atoms and the probability of finding one is mutually exclusive of theother, the probabolity becomes

P =(1 − 1.0 × 10−6

)N(10.45)

Answer: (C)

David S. Latchman ©2009

Page 75: 48578900 GRE Physics 8677 Solutions

DRAFT

The Muon 7510.16 The Muon

It helps knowing what these particles are

Muon The muon, is a lepton, like the electron. It has the ame charge, −e and spin, 1/2,as the electron execpt it’s about 200 times heavier. It’s known as a heavy electron.

Electron This is the answer.

Graviton This is a hypothetical particle that mediates the force of gravity. It has nocharge, no mass and a spin of 2. Nothing like an electron.

Photon The photon is the quantum of the electromagnetic field. It has no charge ormass and a spin of 1. Again nothing like an electron.

Pion The Pion belongs to the meson family. Again, nothing like leptons.

Proton This ia a sub atomic particle and is found in the nucleus of all atoms. Nothinglike an electron.

Answer: (A)

10.17 Radioactive Decay

From the changes in the Mass and Atomic numbers after the subsequent decays, weexpect an α and β decay.

Alpha DecayAZX→A−4

Z−2 X′ +42 α (10.46)

Beta DecayAZX→A

Z+1 X′ +−1 e− + υe (10.47)

Combining both gives

AZX→A−4

Z−2 X′ +42 α→

AZ−1 Y +−1 e− + υe (10.48)

Answer: (B)

10.18 Schrodinger’s Equation

We recall that Schrodinger’s Equation is

Eψ = −~2

2m∂2ψ

∂x2 + V(x)ψ (10.49)

Given that

ψ(x) = A exp−

b2x2

2

(10.50)

©2009 David S. Latchman

Page 76: 48578900 GRE Physics 8677 Solutions

DRAFT

76 GR8677 Exam SolutionsWe differentiate and get

∂2ψ

∂x2 =(b4x2− b2

)ψ (10.51)

Plugging into Schrodinger’s Equation, Equation 10.49, gives us

Eψ = −~2

2m

(b4x2− b2

)ψ + V(x)ψ (10.52)

Applying the boundary condition at x = 0 gives

Eψ = −~2

2mb2ψ (10.53)

This gives

−~2b2

2mψ = −

~2

2m

(b4x2− b2

)ψ + v(x)ψ (10.54)

Solving for V(x) gives

V(x) =~2b4x2

2m(10.55)

Answer: (B)

10.19 Energy Levels of Bohr’s Hydrogen Atom

We recall that the Energy Levels for the Hydrogen atom is

En = −Z2

n2 13.6 eV (10.56)

where Z is the atomic number and n is the quantum number. This can easily be reducedto

En = −An2 (10.57)

Answer: (E)

10.20 Relativistic Energy

The Rest Energy of a particle is given

E = mc2 (10.58)

The Relativistic Energy is for a relativistic particle moving at speed v

E = γvmc2 (10.59)

We are told that a kaon moving at relativistic speeds has the same energy as the restmass as a proton. Thus

EK+ = Ep (10.60)

David S. Latchman ©2009

Page 77: 48578900 GRE Physics 8677 Solutions

DRAFT

Space-Time Interval 77where

EK+ = γvmK+c2 (10.61)Ep = mpc2 (10.62)

Equating both together gives

γv =mp

mK+

(10.63)

=939494

(10.64)

≈940500 (10.65)

This becomes

γv ≈ 1.9 (10.66)

Solving gives

v2 =2.613.61

c2 (10.67)

This gives v2 in the order of 0.7. Squaring will give an answer that’s greater than 0.7,the only answer is 0.85c.

Answer: (E)

10.21 Space-Time Interval

We recall the Space-Time Interval from section 8.10.

(∆S)2 = (∆x)2 +(∆y

)2+ (∆z)2

− c2 (∆t)2 (10.68)

We get

∆S2 = (5 − 3)2 + (3 − 3)2 + (3 − 1)2− c2(5 − 3)2

= 22 + 02 + 22− 22

= 22

∆S = 2 (10.69)

Answer: (C)

10.22 Lorentz Transformation of the EM field

Lorentz transformations show that electric and magnetic fields are different aspects ofthe same force; the electromagnetic force. If there was one stationary charge in ourrest frame, we would observe an electric field. If we were to move to a moving frameof reference, Lorentz transformations predicts the presence of an additional magneticfield.

Answer: (B)

©2009 David S. Latchman

Page 78: 48578900 GRE Physics 8677 Solutions

DRAFT

78 GR8677 Exam Solutions10.23 Conductivity of a Metal and Semi-Conductor

More of a test of what you know.

A Copper is a conductor so we expect its conductivity to be much greater than that ofa semiconductor. TRUE.

B As the temperature of the conductor is increased its atoms vibrate more and disruptthe flow of electrons. As a result, resistance increases. TRUE.

C Different process. As temperature increases, electrons gain more energy to jump theenergy barrier into the conducting region. So conductivity increases. TRUE.

D You may have paused to think for this one but this is TRUE. The addition of animpurity causes an increase of electron scattering off the impurity atoms. As aresult, resistance increases.5

E The effect of adding an impurity on a semiconductor’s conductivity depends onhow many extra valence electrons it adds or subtracts; you can either widen ornarrow the energy bandgap. This is of crucial importance to electronics today.So this is FALSE.

Answer: (E)

10.24 Charging a Battery

The Potential Difference across the resistor, R is

PD = 120 − 100 = 20 V (10.70)

The Total Resistance is

R + r =VI

=2010

R + 1 = 2⇒ R = 1Ω (10.71)

Answer: (C)

10.25 Lorentz Force on a Charged Particle

We are told that the charged particle is released from rest in the electric and magneticfields. The charged particle will experience a force from the magnetic field only when

5There are one or two cases where this is not true. The addition of Silver increases the conductivityof Copper. But the conductivity will still be less than pure silver.

David S. Latchman ©2009

Page 79: 48578900 GRE Physics 8677 Solutions

DRAFT

K-Series X-Rays 79it moves perpendicular to the direction of the magnetic field lines. The particle willmove along the direction of the electric field.

We can also anylize this by looking at the Lorentz Force equation

Fq = q [E + (v × B)] (10.72)

v is in the same direction as B so the cross product between them is zero. We are leftwith

Fq = qE (10.73)

The force is directed along the electrical field line and hence it moves in a straight line.Answer: (E)

10.26 K-Series X-Rays

To calculate we look at the energy levels for the Bohr atom. As the Bohr atom considersthe energy levels for the Hydrogen atom, we need to modify it somewhat

En = Z2eff

1n2

f

−1n2

i

13.6 eV (10.74)

where Zeff is the effective atomic number and n f and ni are the energy levels. For then f = 1 transition

Zeff = Z − 1 (10.75)

where Z = 28 for nickle. As the electrons come in from ni = ∞, Equation 10.74 becomes

E1 = (28 − 1)2[ 112 −

1∞2

]13.6 eV (10.76)

This works out to

E1 = (272)13.6 eV

≈ (30)2× 13.6 eV (10.77)

This takes us in the keV range.Answer: (D)

10.27 Electrons and Spin

It helps if you knew some facts here.

A The periodic table’s arrangement of elements tells us about the chemical propertiesof an element and these properties are dependent on the valent electrons. Howthese valent electrons are arranged is, of course, dependent on spin. So thischoice is TRUE.

©2009 David S. Latchman

Page 80: 48578900 GRE Physics 8677 Solutions

DRAFT

80 GR8677 Exam SolutionsB The energy of an elecron is quantized and obey the Pauli’s Exclusion Principle. All

the electrons are accomodated from the lowest state up to the Fermi Level and thedistribution among levels is described by the Fermi distribution function, f (E),which defines the probability that the energy level, E, is occupied by an electron.

f (E) =

1, E < EF

0, E > EF

where f (E) is the Fermi-Dirac Distribution

f (E) =1

eE−EF/kT + 1(10.78)

As a system goes above 0K, thermal energy may excite to higher energy statesbut this energy is not shared equally by all the electrons. The way energy isdistributed comes about from the exclusion principle, the energy an electron myabsorb at room temperture is kT which is much smaller than EF = 5eV. We candefine a Fermi Temperature,

EF = kTF (10.79)

which works out to be, TF = 60000K. Thus only electrons close to this temperaturecan be excited as the levels above EF are empty. This results in a small numberof electrons being able to be thermally excited and the low electronic specificheat.[1]

C =π2

2Nk

(TT f

)where kT << EF

So this choice is also TRUE.

C The Zeeman Effect describes what happens to Hydrogen spectral lines in a magneticfield; the spectral lines split. In some atoms, there were further splits in spctrallines that couln’t be explained by magnetic dipole moments. The explanation forthis additional splitting was discovered to be due to electron spin.6

D The deflection of an electron in a uniform magnetic field deflects only in one wayand demonstrates none of the electron’s spin properties. Electrons can be de-flected depending on their spin if placed in a non-uniform magnetic field, as wasdemonstrated in the Stern-Gerlach Experiment.7

E When the Hydrogen spectrum is observed at a very high resolution, closely spaceddoublets are observed. This was one of the first experimental evidence for electronspin.8

6Write up on Zeeman and anomalous Zeemen effects7Write up on Stern-Gerlach Experiment8Write up on Fine Structure

David S. Latchman ©2009

Page 81: 48578900 GRE Physics 8677 Solutions

DRAFT

Normalizing a wavefunction 8110.28 Normalizing a wavefunction

We are givenψ(φ) = Aeimφ (10.80)

Normalizing a function means ∫∞

−∞

|Ψ(x)|2 dx = 1 (10.81)

In this case, we want ∫ 2π

0

∣∣∣ψ(φ)∣∣∣2 dφ = 1 (10.82)

and that ∣∣∣ψ(x)∣∣∣2 = ψ∗(x)ψ(x) (10.83)

So

∣∣∣ψ(φ)∣∣∣2 = A2eimφe−imφ

A2∫ 2π

0dφ = 1

A2 [2π − 0] = 1

⇒ A =1√

2π(10.84)

10.29 Right Hand Rule

First we use the ‘Grip’ rule to tell what direction the magnetic field lines are going.Assuming the wire and current are coming out of the page, the magnetic field is in aclockwise direction around the wire. Now we can turn to Fleming’s Right Hand Rule,to solve the rest of the question.

As we want the force acting on our charge to be parallel to the current direction,we see that this will happen when the charge moves towards the wire9.

Answer: (A)

10.30 Electron Configuration of a Potassium atom

We can alalyze and eliminate

A The n = 3 shell has unfilled d-subshells. So this is NOT TRUE.

B The 4s subshell only has one electron. The s subshell can ‘hold’ two electrons so thisis also NOT TRUE.

C Unknown.9Don’t forget to bring your right hand to the exam

©2009 David S. Latchman

Page 82: 48578900 GRE Physics 8677 Solutions

DRAFT

82 GR8677 Exam SolutionsD The sum of all the electrons, we add all the superscripts, gives 19. As this is a

ground state, a lone potassium atom, we can tell that the atomic number is 19.So this is NOT TRUE.

E Potassium has one outer electron, like Hydrogen. So it will also have a sphericallysymmetrical charge distribution.

10.31 Photoelectric Effect I

We are given|eV| = hυ −W (10.85)

We recall that V is the stopping potential, the voltage needed to bring the current tozero. As electrons are negatively charged, we expect this voltage to be negative.

Answer: (A)

10.32 Photoelectric Effect II

Some history needs to be known here. The photoelectric effect was one of the exper-iments that proved that light was absorbed in discreet packets of energy. This is theexperimental evidence that won Einstein the Nobel Prize in 1921.

Answer: (D)

10.33 Photoelectric Effect III

The quantity W is known as the work function of the metal. This is the energy that isneeded to just liberate an electron from its surface.

Answer: (D)

10.34 Potential Energy of a Body

We recall that

F = −dUdx

(10.86)

Given thatU = kx4 (10.87)

The force on the body becomes

F = −d

dxkx4

= −4kx3 (10.88)

Answer: (B)

David S. Latchman ©2009

Page 83: 48578900 GRE Physics 8677 Solutions

DRAFT

Hamiltonian of a Body 8310.35 Hamiltonian of a Body

The Hamiltonian of a body is simply the sum of the potential and kinetic energies.That is

H = T + V (10.89)

where T is the kinetic energy and V is the potential energy. Thus

H =12

mv2 + kx4 (10.90)

We can also express the kinetic energy in terms of momentum, p. So

H =p2

2m+ kx4 (10.91)

Answer: (A)

10.36 Principle of Least Action

Hamilton’s Principle of Least Action10 states

Φ =

∫T

(T(q(t), q(t)

)− V

(q(t)

))dt (10.92)

where T is the kinetic energy and V is the potential energy. This becomes

Φ =

∫ t2

t1

(12

mv2− kx4

)dt (10.93)

Answer: (A)

10.37 Tension in a Conical Pendulum

This is a simple case of resolving the horizontal and vertical components of forces. Sowe have

T cosθ = mg (10.94)T sinθ = mrω2 (10.95)

Squaring the above two equations and adding gives

T2 = m2g2 + m2r2ω4 (10.96)

Leaving us withT = m

(g2 + r2ω4

)(10.97)

Answer: (E)

©2009 David S. Latchman

Page 84: 48578900 GRE Physics 8677 Solutions

DRAFT

84 GR8677 Exam Solutions

Table 10.1: Truth Table for OR-gate

Input 1 Input 2 Output0 0 00 1 11 0 11 1 1

10.38 Diode OR-gate

This is an OR gate and can be illustrated by the truth table below.Answer: (A)

10.39 Gain of an Amplifier vs. Angular Frequency

We are given that the amplifier has some sort of relationship where

G = Kωa (10.98)

falls outside of the amplifier bandwidth region. This is that ‘linear’ part of the graphon the log-log graph. From the graph, we see that, G = 102, for ω = 3 × 105 second-1.Substituting, we get

102 = K(3 × 105

)a

∴ log(102) = a log[K

(3.5 × 105

)]⇒ a ≈ 2 − 5 (10.99)

We can roughly estimate by subtracting the indices. So our relationship is of the form

G = Kω−2 (10.100)

Answer: (E)

10.40 Counting Statistics

We recall from section 9.4 , that he standard deviation of a counting rate is σ =√

N,where N is the number of counts. We have a count of N = 9934, so the standarddeviation is

σ =√

N =√

9934

10000= 100 (10.101)

Answer: (A)10Write something on this

David S. Latchman ©2009

Page 85: 48578900 GRE Physics 8677 Solutions

DRAFT

Binding Energy per Nucleon 8510.41 Binding Energy per Nucleon

More of a knowledge based question. Iron is the most stable of all the others.11

Answer: (C)

10.42 Scattering Cross Section

We are told the particle density of our scatterer is ρ = 1020 nuclei per cubic centimeter.Given the thickness of our scatterer is ` = 0.1 cm, the cross sectional area is

ρ =NV

=NA`

⇒ A =Nρ`

(10.102)

Now the probability of striking a proton is 1 in a million. So

10.43 Coupled Oscillators

There are two ways this system can oscillate, one mass on the end moves a lot and theother two move out of in the opposite directions but not as much or the centermasscan be stationary and the two masses on the end move out of phase with each other. Inthe latter case, as there isn’t any energy transfer between the masses, the period wouldbe that of a single mass-spring system. The frequency of this would simply be

f =1

√km

(10.103)

where k is the spring constant and m is the mass.Answer: (B)

10.43.1 Calculating the modes of oscillation

In case you require a more rigorous approach, we can calculate the modes of oscillation.The Lagrangian of the system is

L = T − V

=12

m[x2

1 + 2x22 + x2

3

]−

12

k[(x2 − x2)2 + (x3 − x2)2

](10.104)

The equation of motion can be found from

ddt

(∂L∂xn

)=∂L∂xn

(10.105)

11Write up on Binding Energy

©2009 David S. Latchman

Page 86: 48578900 GRE Physics 8677 Solutions

DRAFT

86 GR8677 Exam SolutionsThe equations of motion are

mx1 = k (x2 − x1) (10.106)2mx2 = kx1 − 2kx2 + kx3 (10.107)mx3 = −k (x3 − x2) (10.108)

The solutions of the equations are

x1 = A cos(ωt) x2 = B cos(ωt) x3 = C cos(ωt)x1 = −ω2x1 x2 = −ω2x2 x3 = −ω2x3

(10.109)

Solving this, we get (k −mω2

)x1 − kx2 = 0 (10.110)

−kx1 +(2k − 2mω2

)x2 − kx3 = 0 (10.111)

−kx2 +(k −mω2

)x3 = 0 (10.112)

We can solve the modes of oscillation by solving∣∣∣∣∣∣∣∣k −mω2

−k 0−k 2k − 2mω2

−k0 −k k −mω2

∣∣∣∣∣∣∣∣ = 0 (10.113)

Finding the determinant results in(k −mω2

) [2(k −mω2

)2− k2

]− k

[k(k −mω2

)](10.114)

Solving, we get

ω =km

;km±

√2k

m(10.115)

Substituting ω = k/m into the equations of motion, we get

x1 = −x3 (10.116)x2 = 0 (10.117)

We see that the two masses on the ends move out of phase with each other and themiddle one is stationary.

10.44 Collision with a Rod

Momentum will be conserved, so we can say

mv = MV

V =mvM

(10.118)

Answer: (A)

David S. Latchman ©2009

Page 87: 48578900 GRE Physics 8677 Solutions

DRAFT

Compton Wavelength 8710.45 Compton Wavelength

We recall from subsection 7.8.2, the Compton Equation from Equation 7.33

∆λ = λ′ − λ =h

mec(1 − cosθ) (10.119)

Let θ = 90, we get the Compton Wavelength

λc =h

mec= 2.427 × 10−12m (10.120)

Answer: (C)

10.46 Stefan-Boltzmann’s Equation

We recall the Stefan-Boltzmann’s Equation, Equation 5.13

P(T) = σT4 (10.121)

At temperature, T1,P1 = σT1 = 10 mW (10.122)

We are given T2 = 2T1, so

P2 = σT42

= σ (2T1)4

= 16T42

= 16P1 = 160 mW (10.123)

Answer: (E)

10.47 Franck-Hertz Experiment

The Franck-Hertz Experiment as seen in subsection 7.9.3 deals with the manner inwhich electrons of certain energies scatter or collide with Mercury atoms. At certainenergy levels, the Mercury atoms can ‘absorb’ the electrons energy and be excited andthis occurs in discreet steps.

Answer: (C)

10.48 Selection Rules for Electronic Transitions

We recall the selection rules for photon emission

∆` = ±1 Orbital angular momentum∆m` = 0,±1 Magnetic quantum number∆ms = 0 Secondary spin quantum number,∆ j = 0,±1 Total angular momentum

©2009 David S. Latchman

Page 88: 48578900 GRE Physics 8677 Solutions

DRAFT

88 GR8677 Exam SolutionsNOT FINISHED

Answer: (D)

10.49 The Hamilton Operator

The time-independent Schrodinger equation can be written

Hψ = Eψ (10.124)

We can determine the energy of a quantum particle by regarding the classical nonrel-ativistic relationship as an equality of expectation values.

〈H〉 =

⟨p2

2m

⟩+ 〈V〉 (10.125)

We can solve this through the substition of a momentum operator

p→~

i∂∂x

(10.126)

Substituting this into Equation 10.125 gives us

〈H〉 =

∫ +∞

−∞

ψ∗[−~

2m∂2

∂x2ψ + V(x)ψ]

dx

=

∫ +∞

−∞

ψ∗i~∂∂tψdx (10.127)

So we can get a Hamiltonian operator

H→ i~∂∂t

(10.128)

Answer: (B)

10.50 Hall Effect

The Hall Effect describes the production of a potential difference across a currentcarrying conductor that has been placed in a magnetic field. The magnetic field isdirected perpendicularly to the electrical current.

As a charge carrier, an electron, moves through the conductor, the Lorentz Forcewill cause a deviation in the carge carrier’s motion so that more charges accumulatein one location than another. This asymmetric distribution of charges produces anelectric field that prevents the build up of more electrons. This ‘equilibrium’ voltageacross the conductor is known as the Hall Voltage and remains as long as a currentflows through our conductor.

As the deflection and hence, the Hall Voltage, is determined by the sign of thecarrier, this can be used to measure the sign of charge carriers.

David S. Latchman ©2009

Page 89: 48578900 GRE Physics 8677 Solutions

DRAFT

Debye and Einstein Theories to Specific Heat 89An equilibrium condition is reached when the electric force, generated by the accu-

mulated charge carriers, is equal the the magnetic force, that causes the accumulationof charge carriers. Thus

Fm = evdB Fe = eE (10.129)

The current through the conductor is

I = nAvde (10.130)

For a conductor of width, w and thickness, d, there is a Hall voltage across the widthof the conductor. Thus the electrical force becomes

Fe = eE

=EVH

w(10.131)

The magnetic force is

Fm =BI

neA(10.132)

Equation 10.131 is equal to Equation 10.132, thus

eVH

w=

BInewd

∴ VH =BIned

(10.133)

So for a measured magnetic field and current, the sign of the Hall voltage gives is thesign of the charge carrier.

Answer: (C)

10.51 Debye and Einstein Theories to Specific Heat

The determination of the specific heat capacity was first deermined by the Law ofDulong and Petite. This Law was based on Maxwell-Boltzmann statistics and wasaccurate in its predictions except in the region of low temperatures. At that point thereis a departure from prediction and measurements and this is where the Einstein andDebye models come into play.

Both the Einstein and Debye models begin with the assumption that a crystal ismade up of a lattice of connected quantum harmonic oscillators; choice B.

The Einstein model makes three assumptions

1. Each atom is a three-dimensional quantum harmonic oscillator.

2. Atoms do not interact with each other.

3. Atoms vibrate with the same frequency.

©2009 David S. Latchman

Page 90: 48578900 GRE Physics 8677 Solutions

DRAFT

90 GR8677 Exam SolutionsEinstein assumed a quantum oscillator model, similar to that of the black body radi-ation problem. But despite its success, his theory predicted an exponential decress inheat capacity towards absolute zero whereas experiments followed a T3 relationship.This was solved in the Debye Model.

The Debye Model looks at phonon contribution to specific heat capacity. Thistheory correctly predicted the T3 proportionality at low temperatures but suffered atintemediate temperatures.

Answer: (B)

10.52 Potential inside a Hollow Cube

By applying Gauss’ Law and drawing a Gaussian surface inside the cube, we see thatno charge is enclosed and hence no electric field12. We can realte the electric field tothe potential

E = −∇V (10.134)

Where V is the potential.Gauss’ Law shows that with no enclosed charge we have no electric field inside our

cube. ThusE = −∇V = 0 (10.135)

As Equation 10.134 is equal to zero, the potential is the same throughout the cube.13

Answer: (E)

10.53 EM Radiation from Oscillating Charges

As the charge particle oscillates, the electric field oscillates as well. As the field oscillatesand changes, we would expect this changing field to affect a distant charge. If weconsider a charge along the xy-plane, looking directly along the x-axis, we won’t “see”the charge oscillating but we would see it clearly if we look down the y-axis. If wewere to visualize the field, it would look like a doughnut around the x-axis. Based onthat analysis, we choose (C)

Answer: (C)

10.54 Polarization Charge Density

D = ε0E + P (10.136)

∇ ·D = ε0∇ · E + ∇ · P

=εD∇ · Eκ

− σp

12Draw Cube at potential V with Gaussian Surface enclosing no charge13As we expect there to be no Electric Field, we must expect the potential to be the same throughout

the space of the cube. If there were differences, a charge place inside the cube would move.

David S. Latchman ©2009

Page 91: 48578900 GRE Physics 8677 Solutions

DRAFT

Kinetic Energy of Electrons in Metals 91Answer: (E)14

10.55 Kinetic Energy of Electrons in Metals

Electrons belong to a group known as fermions15 and as a result obey the Pauli Exclu-sion Principle16. So in the case of a metal, there are many fermions present each witha different set of quantum numbers. The electron with the highest energy state is hasan energy value known as the Fermi Energy.

NOT FINSIHEDAnswer: (B)

10.56 Expectation or Mean Value

This is a definition question. The question states that for an operator Q,

〈Q〉 =

∫ +∞

−∞

ψ∗Qψdx (10.137)

This is the very definition of the expectation or mean value of Q.Answer: (C)

10.57 Eigenfuction of Wavefunction

We are given the momentum operator as

p = −i~∂∂x

(10.138)

With an eigenvalue of ~k. We can do this by trying each solution and seeing if theymatch17

− i~∂ψ

∂x= ~kψ (10.139)

A: ψ = cos kx We expect ψ, to have the form of an exponential function. Substitutingthis into the eigenfuntion, Equation 10.139, we have

−i~∂∂x

cos kx = −i~ (−k sin kx)

= i~k sin kx , ~kψ

ψ does not surive our differentiation and so we can eliminate it.14Check Polarization in Griffiths15Examples of fermions include electrons, protons and neutrons16The Pauli Exclusion Principle states that no two fermions may occupy the same quantum state17We can eliminate choices (A) & (B) as we would expect the answer to be an exponential function in

this case. These choices were just done for illustrative purposes and you should know to avoid them inthe exam.

©2009 David S. Latchman

Page 92: 48578900 GRE Physics 8677 Solutions

DRAFT

92 GR8677 Exam SolutionsB: ψ = sin kx This is a similar case to the one above and we can eliminate for this

reason.

−i~∂∂x

sin kx = −i~ (k cos kx)

= −i~k cos kx , ~kψ

Again we see that ψ does not survive when we apply our operator and so we caneliminate this choice as well.

C: ψ = exp−ikx Substituting this into Equation 10.139, gives

−i~∂∂x

e−ikx = −i~(−ike−ikx

)= −~ke−ikx , ~kψ

Close but we are off, so we can eliminate this choice as well.

D: ψ = exp ikx If the above choice didn’t work, this might be more likely to.

−i~∂∂x

eikx = −i~(ikeikx

)= ~ke−ikx = ~kψ

Success, this is our answer.

E: = ψ = exp−kx

−i~∂∂x

e−kx = −i~(−ke−kx

)= −i~ke−kx , ~kψ

Again this choice does not work, so we can eliminate this as well

Answer: (D)

10.58 Holograms

The hologram is an image that produces a 3-dimensional image using both the Am-plitude and Phase of a wave. Coherent, monochromatic light, such as from a laser, issplit into two beams. The object we wish to “photograph” is placed in the path of theillumination beam and the scattered light falls on the recording medium. The secondbeam, the reference beam is reflected unimpeded to the recording medium and thesetwo beams produces an interference pattern.

The intensity of light recorded on our medium is the same as the scattered light fromour object. The interference pattern is a result of phase changes as light is scattered offour object. Thus choices (I) and (II) are true.

Answer: (B)

David S. Latchman ©2009

Page 93: 48578900 GRE Physics 8677 Solutions

DRAFT

Group Velocity of a Wave 9310.59 Group Velocity of a Wave

We are given the dispersion relationship of a wave as

ω2 =(c2k2 + m2

) 12 (10.140)

The Group Velocity of a Wave is

vg =dωdk

(10.141)

By differentiating Equation 10.140 with respect to k, we can determine th group velocity

2ωdω = 2c2kdk

⇒dωdk

=c2kω

=c2k

c2k2 + m2(10.142)

We want to examine the cases as k→ 0 and k→∞.As k→ 0, we have

dωdk

=c20

√0 + m2

= 0 (10.143)

As k→∞, c2k2 >> m2 the denominator becomes√

c2k2 + m ≈ c2k2 (10.144)

Replacing the denominator for our group velocity gives

dωdk

=c2kck

= c (10.145)

Answer: (E)

10.60 Potential Energy and Simple Harmonic Motion

We are given a potential energy of

V(x) = a + bx2 (10.146)

We can determine the mass’s spring constant, k, from V′′(x)

V′′(x) = 2b = k (10.147)

The angular frequency, ω, is

ω2 =km

=2bm

(10.148)

We see this is dependent on b and m.Answer: (C)

©2009 David S. Latchman

Page 94: 48578900 GRE Physics 8677 Solutions

DRAFT

94 GR8677 Exam Solutions10.61 Rocket Equation I

We recall from the rocket equation that u in this case is the speed of the exaust gasrelative to the rocket.

Answer: (E)

10.62 Rocket Equation II

The rocket equation is

mdvdt

+ udmdt

= 0 (10.149)

Solving this equation becomes

mdv = udm∫ v

0dv = u

∫ m

m0

dmm

v = u ln( mm0

)(10.150)

This fits none of the answers given.Answer: (E)

10.63 Surface Charge Density

This question was solved as ‘The Classic Image Problem’[2]. Below is an alternativemethod but the principles are the same. Instead of determining the electrical potential,as was done by Griffiths, we will find the electrical field of a dipole and determine thesurface charge density using

E =σε0

(10.151)

Our point charge, −q will induce a +q on the grounded conducting plane. Theresulting electrical field will be due to a combination of the real charge and the ‘virtual’induced charge. Thus

E = −Eyj = (E− + E+) j

= 2E−j (10.152)

Remember the two charges are the same, so at any point along the x-axis, or rather ourgrounded conductor, the electrical field contributions from both charges will be thesame. Thus

E− =q

4πεr2 cosθ where cosθ =dr

=qd

4πε0r3 (10.153)

David S. Latchman ©2009

Page 95: 48578900 GRE Physics 8677 Solutions

DRAFT

Maximum Power Theorem 95Our total field becomes

E =2qd

4πε0r3 (10.154)

You may recognize that 2qd is the electrical dipole moment. Now, putting Equa-tion 10.154 equal to Equation 10.151 gives us

σε0

=qd

2πε0r3 (10.155)

where r = D, we get

σ =qd

2πD2 (10.156)

Answer: (C)

10.64 Maximum Power Theorem

We are given the impedance of our generator

Zg = Rg + jXg (10.157)

For the maximum power to be transmitted, the maximum power theorem states that theload impedance must be equal to the complex conjugate of the generator’s impedance.

Zg = Z∗` (10.158)

Thus

Z` = Rg + jX`

= Rg − jXg (10.159)

Answer: (C)

10.65 Magnetic Field far away from a Current carryingLoop

The Biot-Savart Law is

dB =µ0i4π

d` × rr3 (10.160)

©2009 David S. Latchman

Page 96: 48578900 GRE Physics 8677 Solutions

DRAFT

96 GR8677 Exam SolutionsLet θ be the angle between the radius, b and the radius vector, r, we get

B =µ0i4π

rd` cosθr3 where cosθ =

br

=mu0i4π

d` cosθr2

=µ0i4π

bd`r3 where r =

b2 + h2

=µ0i4π

bd`

(b2 + h2)32

where d` = b · dθ

=µ0i4π·

b2

(b2 + h2)32

2π∫0

=µ0i2

b2

(b2 + h2)32

(10.161)

we see thatB ∝ ib2 (10.162)

Answer: (B)

10.66 Maxwell’s Relations

To derive the Maxwell’s Relations we begin with the thermodynamic potentials

First LawdU = TdS − PdV (10.163)

Entalpy

H = E + PV∴ dH = TdS + VdP (10.164)

Helmholtz Free Energy

F = E − TS∴ dF = −SdT − PdV (10.165)

Gibbs Free Energy

G = E − TS + PV∴ dG = −SdT + VdP (10.166)

David S. Latchman ©2009

Page 97: 48578900 GRE Physics 8677 Solutions

DRAFT

Partition Functions 97All of these differentials are of the form

dz =

(∂z∂x

)y

dx +

(∂z∂y

)x

dy

= Mdx + Ndy

For the variables listed, we choose Equation 10.163 and applying the above conditionwe get

T =

(∂U∂S

)V

P =

(∂U∂V

)S

(10.167)

Thus taking the inverse of T, gives us

1T

=

(∂S∂U

)V

(10.168)

Answer: (E)

10.67 Partition Functions

NOT FINISHED

10.68 Particle moving at Light Speed

Answer: (A)

10.69 Car and Garage I

We are given the car’s length in its rest frame to be L′ = 5 meters and its LorentzContracted length to be L = 3 meters. We can determine the speed from Equation 8.3

L = L′√

1 −v2

c2(35

)2

= 1 −v2

c2

⇒ v =45

c (10.169)

Answer: (C)

10.70 Car and Garage II

As the car approaches the garage, the driver will notice that things around him, in-cluding the garage, are length contracted. We have calculated that the speed that

©2009 David S. Latchman

Page 98: 48578900 GRE Physics 8677 Solutions

DRAFT

98 GR8677 Exam Solutionshe is travelling at to be, v = 0.8c, in the previous section. We again use the LengthContraction formula, Equation 8.3, to solve this question.

Lg = L′g

(1 −

v2

c2

)= 4

(1 − 0.82

)= 2.4 meters (10.170)

Answer: (A)

10.71 Car and Garage III

This is more of a conceptual question. What happens depends on whose frame ofreference you’re in.

Answer: (E)

10.72 Refrective Index of Rock Salt and X-rays

No special knowledge is needed but a little knowledge always helps. You can start byeliminating choices when in doubt.

Choice A NOT TRUE Relativity says nothing about whether light is in a vacuum ornot. If anything, this choice goes against the postulates of Special Relativity. Thelaws of Physics don’t change in vacuum.

Choice B NOT TRUE. X-rays can “transmit” signals or energy; any waveform canonce it is not distorted too much during propagation.

Choice C NOT TRUE. Photons have zero rest mass. Though the tachyon, a hypothet-ical particle, has imaginary mass. This allows it to travel faster than the speed orlight though they don’t violate the principles of causality.

Choice D NOT TRUE. How or when we discover physical theories has no bearingon observed properties or behavior; though according to some it may seem so attimes18

Choice E The phase and group speeds can be different. The phase velocity is the rate atwhich the crests of the wave propagate or the rate at which the phase of the waveis moving. The group speed is the rate at which the envelope of the waveform

18There is a quote by Douglas Adams[3],

There is a theory which states that is ever anyone discovers exactly what the Universe isfor and why it is here, it will instantly disappear and be replaced by something even morebizarre and inexplicable.

There is another which states this has already happened.

So maybe the order in which discoveries are made matters. Who am I to question Douglas Adams?

David S. Latchman ©2009

Page 99: 48578900 GRE Physics 8677 Solutions

DRAFT

Thin Flim Non-Reflective Coatings 99is moving or rather it’s the rate at which the amplitude varies in the waveform.We can use this principle of n < 1 materials to create X-ray mirrors using “totalexternal reflection”.

Answer: (E)

10.73 Thin Flim Non-Reflective Coatings

To analyze this system, we consider our lens with refractive index, n3, being coated byour non-reflective coating of refractive index, n2, and thickness, t, in air with refractiveindex, n1, where

n1 < n2 < n3 (10.171)

As our ray of light in air strikes the first boundary, the coating, it moves from a lessoptically dense medium to a more optically dense one. At the point where it reflects,there will be a phase change in the reflected wave. The transmitted wave goes throughwithout a phase change.

The refracted ray passes through our coating to strike our glass lens, which isoptically more dense than our coating. As a result there will be a phase change inour reflected ray. Destructive interference occurs when the optical path difference, 2t,occurs in half-wavelengths multiples. So

2t =(m +

12

)λn2

(10.172)

where m = 0; 1; 2; 3. The thinnest possible coating occurs at m = 0. Thus

t =14λn2

(10.173)

We need a non-reflective coating that has an optical thicknes of a quarter wavelength.Answer: (A)

10.74 Law of Malus

The Law of Malus states that when a perfect polarizer is placed in a polarized beamof light, the intensity I, is given by

I = I0 cos2 θ (10.174)

where θ is the angle between the light’s plane of polarization and the axis of thepolarizer. A beam of light can be considered to be a uniform mix of plane polarizationangles and the average of this is

I = I0

∫ 2π

0cos2 θ

=12

I0 (10.175)

©2009 David S. Latchman

Page 100: 48578900 GRE Physics 8677 Solutions

DRAFT

100 GR8677 Exam SolutionsSo the maximum fraction of transmitted power through all three polarizers becomes

I3 =(12

)3

=I0

8(10.176)

Answer: (B)

10.75 Geosynchronous Satellite Orbit

We can relate the period or the angluar velocity of a satellite and Newton’s Law ofGravitation

mRω2 = mR(2π

T

)2

=GMm

R2 (10.177)

where M is the mass of the Earth, m is the satellite mass and RE is the orbital radius.From this we can get a relationship between the radius of orbit and its period, whichyou may recognize as Kepler’s Law.

R3∝ T2 (10.178)

We can say

R3E ∝ (80)2 (10.179)

R3S ∝ (24 × 60)2 (10.180)

(10.181)

Dividing Equation 10.180 and Equation 10.181, gives(RS

RE

)3

=(24 × 60

80

)2

R3S = 182R3

E (10.182)

Answer: (B)

10.76 Hoop Rolling down and Inclined Plane

As the hoop rolls down the inclined plane, its gravitational potential energy is con-verted to translational kinetic energy and rotational kinetic energy

Mgh =12

Mv2 +12

Iω2 (10.183)

Recall that v = ωR, Equation 10.183 becomes

MgH =12

MR2ω2 +12

(MR2

)ω2 (10.184)

Solving for ω leaves

ω =

(ghR2

) 12

(10.185)

David S. Latchman ©2009

Page 101: 48578900 GRE Physics 8677 Solutions

DRAFT

Simple Harmonic Motion 101The angular momentum is

L = Iω (10.186)

Substituting Equation 10.185 gives us

L = MR(

ghR2

) 12

= MR√

gh (10.187)

Answer: (A)

10.77 Simple Harmonic Motion

We are told that a particle obeys Hooke’s Law, where

F = −kx (10.188)

We can write the equation of motion as

mx − kx where ω2 =km

where

x = A sin(ωt + φ

)(10.189)

and x = ωA cos(ωt + φ

)(10.190)

We are told that12

= sin(ωt + φ

)(10.191)

We can show that

cos(ωt + φ

)=

√3

2(10.192)

Substituting this into Equation 10.190 gives

x = 2π f A ·√

32

=√

3π f A (10.193)

Amswer: (B)

10.78 Total Energy between Two Charges

We are told three things

1. There is a zero potential energy, and

©2009 David S. Latchman

Page 102: 48578900 GRE Physics 8677 Solutions

DRAFT

102 GR8677 Exam Solutions2. one particle has non-zero speed and hence kinetic energy.

3. No radiation is emitted, so no energy is lost.

The total energy of the system is

E = Potential Energy + Kinetic Energy= 0 + (KE > 0)> 0 (10.194)

Applying the three condition, we expect the total energy to be positive and constant.Answer: (C)

10.79 Maxwell’s Equations and Magnetic Monopoles

You may have heard several things about the∇·B = 0 equation in Maxwell’s Laws. Oneof them is there being no magnetic monopoles or charges. There are some implicationsto this. No charge implies that the amount of field lines that enter a Gaussian surfacemust be equal to the amount of field lines that leave. So using this principle we knowfrom the electric form of this law we can get an answer to this question.

Choice A The number of field lines that enter is the same as the number that leaves.So this does not violate the above law.

Choice B Again we see that the number of field lines entering is the same as thenumber leaving.

Choice C The same as above

Choice D In this case, we see that the field lines at the edge of the Gaussian Surfaceare all leaving; no field lines enter the surface. This is also what we’d expect thefield to look like for a region bounded by a magnetic monopole.

Choice E The field loops in on itself, so the total number of field lines is zero. This fitswith the above law.

Answer: (D)

10.80 Gauss’ Law

To determine an electric field that could exist in a region of space with no charges weturn to Gauss’ Law.

∇ · E = 0 (10.195)

or rather∂∂x

Ex +∂∂y

Ey +∂∂z

Ez = 0 (10.196)

So we analyze each choice in turn to get our answer.

David S. Latchman ©2009

Page 103: 48578900 GRE Physics 8677 Solutions

DRAFT

Biot-Savart Law 103Choice A

E = 2xyi − xyk

∇ · E =∂∂x

2xy +∂∂z

(−xz)

= 2y + x , 0 (10.197)

Choice B

E = −xy j + xzk

∇ · E =∂∂y

(−xy) +∂∂z

xz

= −x + x = 0 (10.198)

Choice C

E = xzi + xz j

∇ · E =∂∂x

xz +∂∂y

xz

= z + 0 , 0 (10.199)

Choice D

E = xyz(i + j)

∇ · E =∂∂x

xyz +∂∂y

xyz

= yz + xz , 0 (10.200)

Choice E

E = xyzi

∇ · E =∂∂x

xyz

= yz , 0 (10.201)

Answer: (B)

10.81 Biot-Savart Law

We can determine the magnetic field produced by our outer wire from the Biot-SavartLaw

dB =µ0

4πd` × r

r3 (10.202)

©2009 David S. Latchman

Page 104: 48578900 GRE Physics 8677 Solutions

DRAFT

104 GR8677 Exam SolutionsAs our radius and differential length vectors are orthogonal, the magnetic field worksout to be

dB =µ0

4πId`rr3

=µ0I4π·

rdθr2

B =µ0I4πr

∫ 2π

0dθ

=µ0I2b

(10.203)

We know from Faraday’s Law, a changing magnetic flux induces a EMF,

E =dΦ

dt(10.204)

where Φ = BA. The magnetic flux becomes

Φ =µ0I2b· πa2 (10.205)

The induced EMF becomes

E =µ0π

2

(a2

b

)dIdt

=µ0π

2

(a2

b

)ωI0 sinωt (10.206)

Answer: (B)

10.82 Zeeman Effect and the emission spectrum of atomicgases

Another knowledge based question best answered by the process of elimination.

Stern-Gerlach Experiemnt The Stern-Gerlach Experiment has nothing to do withspectral emissions. This experiment, performed by O. Stern and W. Gerlachin 1922 studies the behavior of a beam atoms being split in two as they passthrough a non-uniform magnetic field.

Stark Effect The Stark Effect deals with the shift in spectral lines in the presence ofelectrical fields; not in magnetic fields.

Nuclear Magnetic Moments of atoms Close, the splitting seen in the Stern-GerlachExperiment is due to this. Emission spectrum typically deals with electrons andso we would expect it to deal with electrons on some level.

David S. Latchman ©2009

Page 105: 48578900 GRE Physics 8677 Solutions

DRAFT

Spectral Lines in High Density and Low Density Gases 105Emission lines are split in two Closer but still not accurate. There is splitting but in

some cases it may be more than two.

Emission lines are greater or equal than in the absence of the magnetic field This weknow to be true.

The difference in the emission spectrum of a gas in a magnetic field is due to theZeeman effect.

Answer: (E)

10.83 Spectral Lines in High Density and Low DensityGases

We expect the spectral lines to be broader in a high density gas and narrower in a lowdensity gas ue to the increased colissions between the molecules. Atomic collisionsadd another mechanism to transfer energy.[4]

Answer: (C)

10.84 Term Symbols & Spectroscopic Notation

To determine the term symbol for the sodium ground state, we start with the electronicconfiguration. This is easy as they have given us the number of electrons the elementhas thus allowing us to fill sub-shells using the Pauli Exclusion Principle. We get

1s2, 2s2, 2p6, 3s1 (10.207)

We are most interested in the 3s1 sub-shell and can ignore the rest of the filled sub-shells. As we only have one valence electron then ms = +1/2. Now we can calculatethe total spin quantum number, S. As there is only one unpaired electron,

S =12

(10.208)

Now we can calculate the total angular momentum quantum number, J = L + S. Asthe 3s sub-shell is half filled then

L = 0 (10.209)

This gives us

J =12

(10.210)

and as L = 0 then we use the symbol S. Thus our term equation becomes

2S 12

(10.211)

Answer: (B)

©2009 David S. Latchman

Page 106: 48578900 GRE Physics 8677 Solutions

DRAFT

106 GR8677 Exam Solutions10.85 Photon Interaction Cross Sections for Pb

Check Brehm p. 789Answer: (B)

10.86 The Ice Pail Experiment

Gauss’ law is equivalent to Coulomb’s Law because Coulomb’s Law is an inversesquare law; testing one is a valid test of the other. Much of our knowledge of theconsequences of the inverse square law came from the study of gravity. Jason Priestlyknew that there is no gravitational field within a spherically symmetrical mass distri-bution. It was suspected that was the same reason why a charged cork ball inside acharged metallic container isn’t attracted to the walls of a container.

Answer: (E)

10.87 Equipartition of Energy and Diatomic Molecules

To answer this question, we will turn to the equipartition of energy equation

cv =

(f2

)R (10.212)

where f is the number of degrees of freedom. In the case of Model I, we see that So the

Table 10.2: Specific Heat, cv for a diatomic molecule

Degrees of Freedom Model I Model IITranslational 3 3Rotational 2 2Vibrational 0 2Total 5 7

specific heats for Models I & II are

cvI =52

Nk cvII =72

Nk

Now we can go about choosing our answer

Choice A From our above calculations, we see that cvI = 5/2Nk. So this choice isWRONG.

Choice B Again, our calculations show that the specific heat for Model II is larger thanthan of Model I. This is due to the added degrees of freedom (vibrational) that itpossesses. So this choice is WRONG.

David S. Latchman ©2009

Page 107: 48578900 GRE Physics 8677 Solutions

DRAFT

Fermion and Boson Pressure 107C & D They both contradict the other and they both contradict Choice (E).

E This is TRUE. We know that at higher temperatures we have an additional degreeof freedom between our diatomic molecule.

Answer: (E)

10.88 Fermion and Boson Pressure

To answer this question, we must understand the differences between fermions andbosons. Fermions follow Fermi-Dirac statistics and their behavior is obey the PauliExclusion Principle. Basically, this states that no two fermions may have the samequantum state. Bosons on the other hand follow Bose-Einstein statistics and severalbosons can occupy the same quantum state.

As the temperature of a gas drops, the particles are going to fill up the availableenergy states. In the case of fermions, as no two fermions can occupy the same state,then these particles will try to occupy all the energy states it can until the highest isfilled. Bosons on the other hand can occupy the same state, so they will all ‘group’together for the lowest they can. Classically, we don’t pay attention to this grouping,so based on our analyis, we expect,

PF > PC > PB (10.213)

where PB is the boson pressure, PC is the pressure with no quantum effects taking placeand PF to be the fermion pressure.

Answer: (B)

10.89 Wavefunction of Two Identical Particles

We are given the wavefunction of two identical particles,

ψ =1√

2

[ψα(x1)ψβ(x2) + ψβ(x1)ψα(x2)

](10.214)

This is a symmetric function and satisfies the relation

ψαβ(x2, x1) = ψαβ(x1, x2) (10.215)

Symmetric functions obey Bose-Einstein statistics and are known as bosons[6, 7, 8].Upon examination of our choices, we see that19

electrons fermion

positrons fermion

protons fermion

19You could have easily played the ‘one of thes things is not like the other...’ game

©2009 David S. Latchman

Page 108: 48578900 GRE Physics 8677 Solutions

DRAFT

108 GR8677 Exam Solutionsneutron fermion

deutrons Boson

Incidentally, a anti-symmetric function takes the form,

ψ =1√

2

[ψα(x1)ψβ(x2) − ψβ(x1)ψα(x2)

](10.216)

and satisfies the relationψαβ(x2, x1) = −ψαβ(x1, x2) (10.217)

These obey Fermi-Dirac Statistics and are known as fermions.Answer: (E)

10.90 Energy Eigenstates

We may recognize this wavefunction from studying the particle in an infinite wellproblem and see this is the n = 2 wavefunction. We know that

En = n2E0 (10.218)

We are given that E2 = 2 eV. So

E0 =1n2 E2

=24

eV

=12

eV (10.219)

Answer: (C)

10.91 Bragg’s Law

We recall Bragg’s Law2d sinθ = nλ (10.220)

Plugging in what we know, we determine λ to be

λ = 2(3Å)(sin 30)

= 2(3Å)(0.5)

= 3Å (10.221)

We employ the de Broglie relationship between wavelength and momentum

p =hλ

(10.222)

David S. Latchman ©2009

Page 109: 48578900 GRE Physics 8677 Solutions

DRAFT

Selection Rules for Electronic Transitions 109We get

mv =hλ

⇒ v =h

=6.63 × 10−34

(9.11 × 10−31)(3 × 10( − 10))(10.223)

We can determine the order of our answer by looking at the relevant indices

− 34 − (−31) − (−10) = 7 (10.224)

We see that (D) is close to what we are looking for.Answer: (D)

10.92 Selection Rules for Electronic Transitions

The selection rules for an electric dipole transition are[9]

∆` = ±1 Orbital angular momentum∆m` = 0,±1 Magnetic quantum number∆ms = 0 Secondary spin quantum number,∆ j = 0,±1 Total angular momentum

We have no selection rules for spin, ∆s, so we can eliminate this choice.Answer: (D)

10.93 Moving Belt Sander on a Rough Plane

We know the work done on a body by a force is

W = F × x (10.225)

We can relate this to the power of the sander; power is the rate at which work is done.So

P =dWdt

= Fdxdt

= Fv (10.226)

The power of the sander can be calculated

P = VI (10.227)

©2009 David S. Latchman

Page 110: 48578900 GRE Physics 8677 Solutions

DRAFT

110 GR8677 Exam Solutionswhere V and I are the voltage across and the current through the sander. By equatingthe Mechanical Power, Equation 10.226 and the Electrical Power, Equation 10.227, wecan determine the force that the motor exerts on the belt.

F =VIv

=120 × 9

10= 108 N (10.228)

The sander is motionless, soF − µR = 0 (10.229)

where R is the normal force of the sander pushing against the wood. Thus the coefficientof friction is

µ =FR

=108100

= 1.08 (10.230)

Answer: (D)

10.94 RL Circuits

When the switch, S, is closed, a magnetic field builds up within the inductor and theinductor stores energy. The charging of the inductor can be derived from Kirchoff’sRules.

E − IR − LdIdt

= 0 (10.231)

and the solution to this is

I(t) = I0

[1 − exp

(R1tL

)](10.232)

where the time constant, τ1 = L/R1.We can find the voltage across the resistor, R1, by multiplying the above by R1,

giving us

V(t) = R1 · I0

[1 − exp

(R1tL

)]= E

[1 − exp

(R1tL

)](10.233)

The potential at A can be found by measuring the voltage across the inductor. Giventhat

E − VR1 − VL = 0∴ VL = E − VR1

= E exp(R1t

L

)(10.234)

David S. Latchman ©2009

Page 111: 48578900 GRE Physics 8677 Solutions

DRAFT

Carnot Cycles 111This we know to be an exponential decay and (fortunately) limits our choices to either(A) or (B)20

The story doesn’t end here. If the inductor was not present, the voltage wouldquickly drop and level off to zero but with the inductor present, a change in currentmeans a change in magnetic flux; the inductor opposes this change. We would expectto see a reversal in the potential at A. Since both (A) and (B) show this flip, we need tothink some more.

The energy stored by the inductor is

UL =12

LI20 =

12

L(E

R1

)2

(10.235)

With S opened, the inductor is going to dump its energy across R2 and assuming thatthe diode has negligible resistance, all of this energy goes to R2. Thus

U =12

L(

VR2

R2

)2

(10.236)

The above two equations are equal, thus

E

R1=

VR2

R2

VR2 = 3E (10.237)

We expect the potential at A to be larger when S is opened. Graph (B) fits this choice.Answer: B

10.95 Carnot Cycles

The Carnot Cycle is made up of two isothermal transformations, KL and MN, and twoadiabatic transformations, LM and NK. For isothermal transformations, we have

PV = nRT = a constant (10.238)

For adiabatic transformations, we have

PVγ = a constant (10.239)

where γ = CP/CV.For the KL transformation, dU = 0.

Q2 = WK→L

∴WK→L =

∫ VL

VK

PdV

= nRT2 ln(VK

VL

)(10.240)

20If you get stuck beyond this point, you can guess. The odds are now in your favor.

©2009 David S. Latchman

Page 112: 48578900 GRE Physics 8677 Solutions

DRAFT

112 GR8677 Exam SolutionsFor the LM transformation,

PLVγL = PMVγ

M (10.241)

For the MN transformation, dU = 0.

Q1 = WM→N

∴WM→N =

∫ VN

VM

PdV

= nRT1 ln(VN

VM

)(10.242)

For the NK transformation,PNVγ

N = PKVγK (10.243)

Dividing Equation 10.241 and Equation 10.243, gives

PLVγL

PKVγK

=PMVγ

M

PNVγN

∴VL

VK=

VM

VN(10.244)

The effeciency of an engine is defined

η = 1 −Q1

Q2(10.245)

We get

η = 1 −Q1

Q2= 1 −

−WM→N

WK→L

= 1 −nRT1 ln

(VMVN

)nRT2 ln

(VKVL

)= 1 −

T1

T2(10.246)

1. We see that

1 −Q1

Q2= 1 −

T1

T2

∴Q1

Q2=

T1

T2(10.247)

Thus choice (A) is true.

2. Heat moves from the hot reservoir and is converted to work and heat. Thus

Q2 = Q1 + W (10.248)

The entropy change from the hot reservoir

S =dQ2

T(10.249)

As the hot reservoir looses heat, the entropy decreases. Thus choice (B) is true.

David S. Latchman ©2009

Page 113: 48578900 GRE Physics 8677 Solutions

DRAFT

First Order Perturbation Theory 1133. For a reversible cycle, there is no net heat flow over the cycle. The change in

entropy is defined by Calusius’s Theorem.∮dQT

= 0 (10.250)

We see that the entropy of the system remains the same. Thus choice (C) is false.

4. The efficieny is defined

η =WQ2

(10.251)

This becomes

η = 1 −Q1

Q2

=Q2 −Q1

Q2(10.252)

Thus W = Q2 −Q1. So choice (D) is true,

5. The effeciency is based on an ideal gas and has no relation to the substance used.So choice (E) is also true.

Answer: (C)

10.96 First Order Perturbation Theory

Perturbation Theory is a procedure for obtaining approximate solutions for a perturbedstate by studying the solutions of the unperturbed state[10]. We can, and shouldn’t,calculate this in the exam.

We can get the first order correction to be ebergy eigenvalue[11]

E1n = 〈ψ0

n|H′

|ψ0n〉 (10.253)

From there we can get the first order correction to the wave function

ψ1n =

∑m,n

〈ψ0m|H

|ψ0n〉(

E0n − E0

m) (10.254)

and can be expressed asψ1

n =∑m,n

c(n)m ψ

0m (10.255)

you may recognize this as a Fourier Series and this will help you knowing that theperturbing potential is one period of a saw tooth wave. And you may recall that theFourier Series of a saw tooth wave form is made up of even harmonics.

Answer: (B)21

21Griffiths gives a similar problem in his text[12]

©2009 David S. Latchman

Page 114: 48578900 GRE Physics 8677 Solutions

DRAFT

114 GR8677 Exam Solutions10.97 Colliding Discs and the Conservation of Angular

Momentum

As the disk moves, it possessed both angular and linear momentums. We can notexactly add these two as they, though similar, are quite different beasts. But we candefine a linear angular motion with respect to some origin. As the two discs hit eachother, they fuse. This slows the oncoming disc. We can calculate the linear angularmomentum

L = r × p (10.256)

where p is the linear momentum and r is the distance from the point P to the center ofdisc I. This becomes

Lv0 = MR × v0

= −MRv0 (10.257)

It’s negative as the cross product of R and v0 is negative.The Rotational Angular Momentum is

Lω0 = Iω0 (10.258)

Adding Equation 10.258 and Equation 10.257 gives the total angular momentum.

L = Lω0 + Lv0

= Iω0 −MRv0

=12

MR2ω0 −12

MR2ω0

= 0

Thus the total angular momentum at the point P is zero.Answer: (A)

10.98 Electrical Potential of a Long Thin Rod

We have charge uniformly distributed along the glass rod. It’s linear charge density is

λ =Q`

=dQdx

(10.259)

The Electric Potential is defined

V(x) =q

4πε0x(10.260)

We can ‘slice’ our rod into infinitesimal slices and sum them to get the potential of therod.

dV =1

4πε0

λdxx

(10.261)

David S. Latchman ©2009

Page 115: 48578900 GRE Physics 8677 Solutions

DRAFT

Ground State of a Positronium Atom 115We assume that the potential at the end of the rod, x = ` is V = 0 and at some pointaway from the rod, x, the potential is V. So∫ V

0dV =

λ4πε0

∫ x

`

dxx

4πε0ln

(x`

)(10.262)

Where x = 2`, Equation 10.262 becomes

V =Q`

14πε0

ln(2``

)=

Q`

14πε0

ln 2 (10.263)

Answer: (D)

10.99 Ground State of a Positronium Atom

Positronium consists of an electron and a positron bound together to form an “exotic”atom. As the masses of the electron and positron are the same, we must use a reduced-mass correction factor to determine the enrgy levels of this system.22. The reducedmass of the system is

=1

me+

1mp

(10.264)

Thus /mu is

µ =me ·mp

me + mp

=me

2(10.265)

The ground state of the Hydrogen atom, in terms of the reduced mass is

E1 = −µ

meE0

= −12

E0 (10.266)

where E0 = 13.6 eV.Answer: (B)

10.100 The Pinhole Camera

A pinhole camera is simply a camera with no lens and a very small aperature. Lightpasses through this hole to produce an inverted image on a screen. For the photography

22Place cite here

©2009 David S. Latchman

Page 116: 48578900 GRE Physics 8677 Solutions

DRAFT

116 GR8677 Exam Solutionsbuffs among you, you know that by varying the size of a camera’s aperature canaccomplish various things; making the aperature bigger allows more light to enter andproduces a “brighter” picture while making the aperature smaller produces a sharperimage.

In the case of the pinhole camera, making the pinhole, or aperature, smaller pro-duces a sharper image because it reduces “image overlap”. Think of a large hole asa set of tiny pinholes places close to each other. This results in an infinite amount ofimages overlapping each other and hence a blurry image. So to produce a sharp image,it is best to use the smallest pinhole possible, the tradeoff being an image that’s not as“bright”.

There are limits to the size of our pinhole. We can not say, for example, use aninfinitely small pinhole the produce the sharpest possible image. Beyond some pointdiffraction effects take place and will ruin our image.

Consider a pinhole camera of length, D, with a pinhole of diameter, d. We knowhow much a beam of light will be diffracted through this pinhole by23

d sinθ = mλ (10.267)

this is the equation for the diffraction of a single slit. As θ is small and we will considerfirst order diffraction effects, Equation 10.267 becomes

dθ = λ

⇒ θ =λd

(10.268)

The “size” of this spread out image is

y = 2θD

=2λD

d(10.269)

So the ‘blur’ of our resulting image is

B = y − d

=2λD

d− d (10.270)

We can see that we want to reduce y as much as possible. i.e. make it d. So Equa-tion 10.270 becomes

0 =2λD

d− d

∴2λD

d= d

Thus d =√

2λD (10.271)

So we’d want a pinhole of that size to produce or sharpest image possible. This resultis close to the result that Lord Rayleigh used, which worked out to be

d = 1.9√

Dλ (10.272)

Answer: (A)23Add image of pinhole camera

David S. Latchman ©2009

Page 117: 48578900 GRE Physics 8677 Solutions

DRAFTAppendix A

Constants & Important Equations

A.1 Constants

Constant Symbol ValueSpeed of light in a vacuum c 2.99 × 108 m/sGravitational Constant G 6.67 × 10−11 m3/kg.s2

Rest Mass of the electron me 9.11 × 10−31 kgAvogadro’s Number NA 6.02 × 1023 mol-1

Universal Gas Constant R 8.31 J/mol.KBoltzmann’s Constant k 1.38 × 10−23 J/KElectron charge e 1.60 × 10−9CPermitivitty of Free Space ε0 8.85 × 10−12 C2/N.m2

Permeability of Free Space µ0 4π × 10−7 T.m/AAthmospheric Pressure 1 atm 1.0 × 105 M/m2

Bohr Radius a0 0.529 × 10−10 m

A.2 Vector Identities

A.2.1 Triple Products

A · (B × C) = B · (C ×A) = C · (A × B) (A.1)A × (B × C) = B (A · C) − C (A · B) (A.2)

Page 118: 48578900 GRE Physics 8677 Solutions

DRAFT

118 Constants & Important EquationsA.2.2 Product Rules

∇(

f g)

= f(∇g

)+ g

(∇ f

)(A.3)

∇ (A · B) = A × (∇ × B) + B × (∇ ×A) + (A · ∇) B + (B · ∇) A (A.4)∇ ·

(f A

)= f (∇ ·A) + A ·

(∇ f

)(A.5)

∇ · (A × B) = B · (∇ ×A) −A · (∇ × B) (A.6)∇ ×

(f A

)= f (∇ ×A) −A ×

(∇ f

)(A.7)

∇ × (A × B) = (B · ∇) A − (A · ∇) B + A (∇ · B) − B (∇ ·A) (A.8)

A.2.3 Second Derivatives

∇ · (∇ ×A) = 0 (A.9)∇ ×

(∇ f

)= 0 (A.10)

∇ × (∇ ×A) = ∇ (∇ ·A) − ∇2A (A.11)

A.3 Commutators

A.3.1 Lie-algebra Relations

[A,A] = 0 (A.12)[A,B] = −[B,A] (A.13)[A, [B,C]] + [B, [C,A]] + [C, [A,B]] = 0 (A.14)

A.3.2 Canonical Commutator

[x, p] = i~ (A.15)

A.3.3 Kronecker Delta Function

δmn =

0 if m , n;1 if m = n;

For a wave function ∫ψm(x)∗ψn(x)dx = δmn (A.16)

A.4 Linear Algebra

A.4.1 Vectors

Vector Addition

The sum of two vectors is another vector

|α〉 + |β〉 = |γ〉 (A.17)

David S. Latchman ©2009

Page 119: 48578900 GRE Physics 8677 Solutions

DRAFT

Linear Algebra 119Commutative

|α〉 + |β〉 = |β〉 + |α〉 (A.18)

Associative|α〉 +

(|β〉 + |γ〉

)=

(|α〉 + |β〉

)+ |γ〉 (A.19)

Zero Vector|α〉 + |0〉 = |α〉 (A.20)

Inverse Vector|α〉 + | − α〉 = |0〉 (A.21)

©2009 David S. Latchman

Page 120: 48578900 GRE Physics 8677 Solutions

DRAFT

120 Constants & Important Equations

David S. Latchman ©2009

Page 121: 48578900 GRE Physics 8677 Solutions

DRAFTBibliography

[1] John J. Brehm and William J. Mullin. Introduction to the Structure of Matter, chapter11-6, pages 567–571. Wiley, first edition, 1989.

[2] David J. Griffiths. Introduction to Electrodyanmics, chapter 3.2.1, pages 121–123.Prentice Hall, third edition, 1999.

[3] Douglas Adams. The restaurant at the end of the universe.

[4] Wikipedia. Spectral line — wikipedia, the free encyclopedia, 2009. [Online;accessed 17-March-2009].

[5] Wikipedia. Term symbol — wikipedia, the free encyclopedia, 2008. [Online;accessed 22-March-2009].

[6] John J. Brehm and William J. Mullin. Introduction to the Structure of Matter, chapter5-10, pages 283–287. Wiley, first edition, 1989.

[7] John J. Brehm and William J. Mullin. Introduction to the Structure of Matter, chapter11-1, pages 539–540. Wiley, first edition, 1989.

[8] David J. Griffiths. Introduction to Quantum Mechanics, chapter 5.1.1, pages 203–205.Prentice Hall, second edition, 2005.

[9] David J. Griffiths. Introduction to Quantum Mechanics, chapter 9.3.3, pages 359–362.Prentice Hall, second edition, 2005.

[10] David J. Griffiths. Introduction to Quantum Mechanics, chapter 6.1.1, page 249.Prentice Hall, second edition, 2005.

[11] David J. Griffiths. Introduction to Quantum Mechanics, chapter 6.1.2, pages 251–254.Prentice Hall, second edition, 2005.

[12] David J. Griffiths. Introduction to Quantum Mechanics, chapter 6.1.2, page 254.Prentice Hall, second edition, 2005.

Page 122: 48578900 GRE Physics 8677 Solutions

DRAFTIndex

AmplifiersGR8677 Q39, 84

Angular Momentum, see Rotational Mo-tion

Archimedes’ Principle, 23

Bernoulli’s Equation, 24Binding Energy

GR8677 Q41, 85Bohr Model

GR8677 Q19, 76Hydrogen Model, 49

Celestial Mechanics, 21Circular Orbits, 22Elliptical Orbit, 23Escape Speed, 22Hyperbolic Orbit, 23Kepler’s Laws, 22Newton’s Law of Gravitation, 21Orbits, 22Parabolic Orbit, 23Potential Energy, 22Vis-viva Equation, 23

Center of Mass, see System of ParticlesCetripetal Motion

GR8677 Q06, 70Circular Orbits, see Celestial MechanicsCommutators, 118

Canonical Commutators, 118Kronecker Delta Function, 118Lie-algebra Relations, 118

Compton Effect, 52Compton Wavelength

GR8677 Q45, 87Conductivity

GR8677 Q23, 78Counting Statistics, 65

GR8677 Q40, 84

Current DensityGR8677 Q09, 71

DielectricsGR8677 Q03, 68

Digital CircuitsGR8677 Q38, 84

Doppler Effect, 19Drag Force

GR8677 Q01, 67

Elastic ColissionsGR8677 Q05, 69

ElectricityGR8677 Q24, 78

Electron SpinGR8677 Q27, 79

Electronic ConfigurationGR8677 Q30, 81

Elliptic Orbits, see OrbitsEnergy

Kinectic Energy, 15Potential Energy, 15Work-Energy Theorem, 15

Equation of Continuity, 24

Fleming’s Right Hand RuleGR8677 Q29, 81

Fluid Dynamics, 23Archimedes’ Principle, 23Bernoulli’s Equation, 24Equation of Continuity, 24

Franck-Hertz Experiment, 55GR8677 Q47, 87

Gauss’ LawGR8677 Q10, 72

Gravitation, see Celestial Mechanics

Hall Effect

Page 123: 48578900 GRE Physics 8677 Solutions

DRAFT

Index 123GR8677 Q50, 88

Hamiltonian, 24GR8677 Q35, 83

Hooke’s Law, 15Potential Energy of a Spring, 15

Hyperbolic Orbits, see Orbits

InterferenceGR8677 Q13, 73

Kepler’s Laws, see Celestial MechanicsKinematics

Circular Motion, 13Linear Motion, 13

Kronecker Delta Function, 118

Laboratory MethodsGR8677 Q40, 84

Lagrangian, 24Linear Algebra, 118

Vectors, 118Lorentz Force Law

GR8677 Q25, 78Lorentz Transformation

GR8677 Q22, 77

Maximum Power TheoremGR8677 Q64, 95

Maxwell’s LawsGR8677 Q11, 73

MechanicsGR8677 Q07, 70GR8677 Q08, 71GR8677 Q37, 83

Moment of Inertia, see Rotational Motion

Newton’s Law of Gravitation, see CelestialMechanics

Newton’s Laws, 14Impulse, 14Momentum, 14

Nuclear PhysicsRadioactive Decay

GR8677 Q17, 75

OrbitsElliptical Orbit, 23Hyperbolic Orbit, 23

Parabolic Orbit, 23Oscillatory Motion, 16

Coupled Harmonic Oscillators, 17GR8677 Q43, 85

Damped Motion, 16Kinetic Energy, 16Potential Energy, 16Simple Harmonic Motion Equation, 16Small Oscillations, 17Total Energy, 16

Parabolic Orbits, see OrbitsParallel Axis Theorem, see Rotational Mo-

tionParticle Physics

MuonGR8677 Q16, 75

Photoelectric EffectGR8677 Q31, 82GR8677 Q32, 82GR8677 Q33, 82

Potential Energy, see EnergyGR8677 Q34, 82

Potential Energy of a Spring, see Hooke’sLaw

Principle of Least ActionGR8677 Q36, 83

ProbabilityGR8677 Q15, 74

Rolling Kinetic Energy, see Rotational Mo-tion

Rotational Kinetic Energy, see RotationalMotion

Rotational Motion, 20Angular Momentum, 20Moment of Inertia, 20Parallel Axis Theorem, 20Rolling Kinetic Energy, 21Rotational Kinetic Energy, 20Torque, 20

Satellite OrbitsGR8677 Q02, 68

Schrodinger’s EquationGR8677 Q18, 75

Space-Time Interval

©2009 David S. Latchman

Page 124: 48578900 GRE Physics 8677 Solutions

DRAFT

124 IndexGR8677 Q21, 77

Special RelativityDoppler Shift

GR8677 Q12, 73Energy

GR8677 Q20, 76Specific Heat

GR8677 Q14, 73Stefan-Boltzmann’s Equation, 40

GR8677 Q46, 87System of Particles, 21

Center of Mass, 21

Thin Film InterferenceGR8677 Q73, 99

Torque, see Rotational Motion

Vector Identities, 117Product Rules, 118Second Derivatives, 118Triple Products, 117

Vis-viva Equation, 23

Wave EquationGR8677 Q04, 68

Wave functionGR8677 Q28, 81

WorkConstant Force, 15

Work-Energy Theorem, see Energy

X-RaysGR8677 Q26, 79

David S. Latchman ©2009